Участник:Iloskutov/Матан 4сем — различия между версиями

Материал из Викиконспекты
Перейти к: навигация, поиск
(Вычисление интеграла Дирихле)
(Теорема о вычислении интеграла по взвешенному образу меры)
 
(не показано 138 промежуточных версий 10 участников)
Строка 1: Строка 1:
 
== Определения ==
 
== Определения ==
 +
=== Условие L_loc ===
 +
{{Определение
 +
|definition=
 +
<tex>\exists U(y_0)</tex> и <tex>\exists g(x)</tex> — суммируемая, что <tex>\forall y \in U(y_0) \quad \forall x : |f(x,y)| \le g(x)</tex><br>
 +
Тогда <tex>f</tex> удовлетворяет <tex>L_{loc}</tex> в точке <tex>y_0</tex>
 +
}}
 +
 +
=== Образ меры при отображении ===
 +
{{Определение
 +
|definition=
 +
Пусть <tex>\Phi^{-1}(\mathfrak B) \subset \mathfrak A</tex><br>
 +
<tex>\nu \colon \mathfrak B \to \overline{\mathbb{R}}, \quad \nu(\mathfrak B) = \mu(\Phi^{-1}(\mathfrak B))</tex> — мера<br>
 +
<tex>\nu</tex> — образ меры <tex>\mu</tex> при отображении <tex>\Phi</tex>
 +
}}
 +
 +
=== Взвешенный образ меры ===
 +
{{Определение
 +
|definition=
 +
<tex>(X, \mathfrak{A}, \mu), \quad (Y, \mathfrak{B}, ?)</tex><br>
 +
<tex>w \geqslant 0</tex> — измеримая на <tex>X</tex> функция<br>
 +
<tex>\Phi \colon X \to Y, \quad \Phi^{-1}(\mathfrak B) \subset \mathfrak A</tex><br>
 +
Тогда <tex dpi=150>\nu(B) = \displaystyle\int\limits_{\Phi^{-1}(\mathfrak B)} w \,d\mu</tex> — взвешенный образ <tex>\mu</tex> при отображении <tex>\Phi</tex>, <tex>w</tex> — вес
 +
}}
 +
 +
=== Плотность одной меры по отношению к другой ===
 +
{{Определение
 +
|definition=
 +
<tex>(X, \mathfrak{A}, \mu), \quad (Y, \mathfrak{B}, \nu) </tex><br>
 +
<tex>X = Y, \quad \mathfrak{A} = \mathfrak{B}, \quad \Phi = id</tex><br>
 +
<tex>w \geqslant 0</tex> — вес, измерим на <tex>X</tex>, <tex>f</tex> — изм. на <tex>X</tex><br>
 +
<tex>\nu(B) = \int\limits_B w(x) d\mu</tex><br>
 +
Тогда <tex>w</tex> — плотность <tex>\nu</tex> относительно <tex>\mu</tex>
 +
}}
 +
 +
=== Заряд ===
 +
{{Определение
 +
|definition=
 +
<tex>(X, \mathfrak{A}, \mu), \quad \mu\colon \mathfrak A \to \mathbb{R}</tex> не обязательно <tex>\geqslant 0</tex> и обладает свойством счётной аддитивности<br>
 +
Тогда <tex>\mu</tex> — заряд
 +
}}
 +
 +
=== Множество положительности заряда ===
 +
{{Определение
 +
|definition=
 +
<tex>\forall E \in B \ (B \in \mathfrak A) \quad \mu E \geqslant 0</tex> (заряд <tex>E</tex> неотрицателен) <br>
 +
<tex>B \in \mathfrak A</tex> — множество положительности
 +
}}
 +
 +
=== Мера, абсолютно непрерывная по отношению к другой мере ===
 +
{{Определение
 +
|definition=
 +
<tex>\mu, \nu \colon \mathfrak A \to \mathbb{R}, \quad \forall a \in \mathfrak A: \mu (a) = 0 \Rightarrow \nu (a) = 0</tex><br>
 +
Тогда <tex>\nu</tex> — абсолютно непрерывная по отношению к мере <tex>\mu</tex>
 +
}}
 +
 +
=== Произведение мер ===
 +
{{Определение
 +
|definition=
 +
<tex>(X, \mathfrak{A}, \mu), \quad (Y, \mathfrak{B}, \nu)</tex><br>
 +
<tex>X \times Y</tex> — декартово произведение, <tex>\mathfrak{A} \times \mathfrak{B} = \{a \times b \mid a \in \mathfrak{A}, b \in \mathfrak{B}\}</tex><br>
 +
<tex>m \colon A \times B \to R^+, \quad m(a \times b) = \mu(a) \cdot \nu(b)</tex><br>
 +
<tex>m</tex> — произведение мер <tex>\mu, \nu</tex> в <tex>(X \times Y, \mathfrak{A} \times \mathfrak{B}, m)</tex>
 +
}}
 +
 +
=== Сечение множества ===
 +
{{Определение
 +
|definition=
 +
Пусть <tex>C \subset X \times Y</tex><br>
 +
<tex>C_x = \{y \in Y | (x, y) \in C\}</tex> - сечение <tex>C</tex> по <tex>X</tex><br>
 +
<tex>C_y = \{x \in X | (x, y) \in C\}</tex> - сечение <tex>C</tex> по <tex>Y</tex>
 +
 +
}}
 +
 +
=== Функция распределения ===
 +
{{Определение
 +
|definition=
 +
<tex>(X, \mathfrak{A}, \mu)</tex><br>
 +
<tex>h: X \to \mathbb{R}, \quad X(h(x) < a)</tex> - конечно<br>
 +
<tex>H(a) = \mu X (h(x) < a)</tex> - функция распределения <tex>(: \mathbb{R} \to \mathbb{R})</tex>
 +
}}
 +
 
=== Интегральные неравенства Гёльдера и Минковского ===
 
=== Интегральные неравенства Гёльдера и Минковского ===
 
{{Теорема
 
{{Теорема
Строка 18: Строка 99:
  
 
=== Интеграл комплекснозначной функции ===
 
=== Интеграл комплекснозначной функции ===
 +
{{Теорема
 +
|statement=
 +
<tex>f \colon \mathbb R \to \overline{\mathbb C}</tex><br>
 +
<tex>(X, \mathfrak A, \mu)</tex>. Тогда:
 +
#<tex>f</tex> — изм., если <tex>\operatorname{Re}(f), \operatorname{Im}(f)</tex> — изм.
 +
#<tex>\displaystyle\int_X f\;d\mu = \int_X \operatorname{Re}(f) \;d\mu + i\int_X \operatorname{Im}(f)\;d\mu</tex><br><!--
 +
--><tex>f</tex> — сумм., <tex>\operatorname{Re}(f), \operatorname{Im}(f)</tex> — сумм.
 +
|proof=
 +
}}
 +
 
=== Пространство $L^p(E,\mu)$ ===
 
=== Пространство $L^p(E,\mu)$ ===
 
{{Определение
 
{{Определение
Строка 24: Строка 115:
  
 
{{Определение
 
{{Определение
|definition=<tex>L^p(E, \mu) = \Bigl\{f \in L^0(E, \mu) \ \Bigm|\ \displaystyle\int_E |f|^p \;d\mu < +\infty \Bigr\}</tex>.
+
|definition=<tex>L^p(E, \mu) = \Bigl\{f \in L^0(E, \mu) \ \Bigm|\ \displaystyle\int\limits_E |f|^p \;d\mu < +\infty \Bigr\}</tex>.
 
}}
 
}}
  
 
=== Пространство $L^\infty(E,\mu)$ ===
 
=== Пространство $L^\infty(E,\mu)$ ===
 
{{Определение
 
{{Определение
|definition=<tex dpi=150>L^\infty(E, \mu) = \left\{ f \in L^0(X, \mu) \ \middle|\  \operatorname*{ess\,sup}\limits_E |f| < +\infty \right\}</tex>
+
|definition=<tex dpi=150>L^\infty(E, \mu) = \Bigl\{ f \in L^0(E, \mu) \ \bigl|\  \operatorname*{ess\,sup}\limits_E |f| < +\infty \Bigr\}</tex>
 
}}
 
}}
  
Строка 46: Строка 137:
  
 
=== Плотное множество ===
 
=== Плотное множество ===
 +
{{Определение
 +
|definition=<tex>X</tex> — метрическое пространство.
 +
 +
<tex>A \subset X</tex> — (всюду) плотно в <tex>X</tex>, если
 +
для любого открытого мн-ва <tex>G \subset X \quad A \cap G \ne \varnothing</tex>.
 +
 +
Или, эквивалентно, любой шар <tex>B(x_0, r)</tex> содержит точки из <tex>A</tex>.
 +
}}
 +
 
=== Финитная функция ===
 
=== Финитная функция ===
 +
{{Определение
 +
|definition=<tex>f</tex> — финитная в <tex>\mathbb R^m</tex>, если она равна нулю вне некоторого шара.
 +
}}
 +
 
=== Гильбертово пространство ===
 
=== Гильбертово пространство ===
 
{{Определение
 
{{Определение
|definition=<tex>\mathcal H</tex> — полное (любая фундаментальная последовательность сходится в этом пространстве) линейное пространство со скалярным произведением
+
|definition=<tex>\mathcal H</tex> — полное (любая фундаментальная последовательность сходится в этом пространстве) линейное пространство со скалярным произведением. Под полнотой понимается полнота относительно метрики, порождённой скалярным произведением.
 
}}
 
}}
 
{{Определение
 
{{Определение
Строка 76: Строка 180:
 
}}
 
}}
 
=== Сходящийся ряд в гильбертовом пространстве ===
 
=== Сходящийся ряд в гильбертовом пространстве ===
 +
{{Определение
 +
|definition=Ряд сходится, если существует элемент из гильбертового
 +
пространства, являющийся пределом частичных сумм.
 +
}}
 +
 
=== Коэффициенты Фурье, ряд Фурье ===
 
=== Коэффициенты Фурье, ряд Фурье ===
 
{{Определение
 
{{Определение
|definition=<tex>t \in L^1[-\pi; \pi]</tex>, тогда <tex>a_k, b_k, c_k</tex> — коэффициенты Фурье для <tex>t (a_k(f), b_k(f), c_k(f))</tex>, а ряд <tex>\dfrac{a_0(t)}{2} + \sum a_k(t) \cos kx + b_k(t) \sin kx \ ; \sum c_k(t) e^{2kt}</tex> — ряд Фурье
+
|definition=<tex>t \in L^1[-\pi; \pi]</tex>, тогда <tex>a_k, b_k, c_k</tex> — коэффициенты Фурье для <tex>t (a_k(f), b_k(f), c_k(f))</tex>, а ряд <tex>\dfrac{a_0(t)}{2} + \sum a_k(t) \cos kx + b_k(t) \sin kx \ ; \sum c_k(t) e^{ikt}</tex> — ряд Фурье
 
}}
 
}}
  
Строка 91: Строка 200:
  
 
=== Тригонометрический ряд ===
 
=== Тригонометрический ряд ===
 +
{{Определение
 +
|definition=<tex>T_n(x) = \dfrac{a_0}{2} + \displaystyle\sum_{k=1}^n a_k \cos kx + b_k \sin kx</tex> — тригонометрический полином степени <tex>n</tex>.
 +
}}
 +
{{Определение
 +
|definition=<tex>T(x) = \dfrac{a_0}{2} + \displaystyle\sum_{k=1}^{+\infty} a_k \cos kx + b_k \sin kx</tex> — тригонометрический ряд.
 +
}}
 +
 
=== Коэффициенты Фурье функции ===
 
=== Коэффициенты Фурье функции ===
 +
{{Определение
 +
|definition= Коэффициенты Фурье функции <tex>f</tex> — <tex>a_0(f), a_k(f), b_k(f), c_k(f)</tex> из формулы тригонометрического ряда.
 +
 +
Можно вычислить по формулам:
 +
<tex>
 +
a_0 = \dfrac{1}{\pi} \cdot \displaystyle\int^\pi_{-\pi} f(x) \,dx \\
 +
a_k = \dfrac{1}{\pi} \cdot \displaystyle\int^\pi_{-\pi} f(x) \cos kx \,dx \\
 +
b_k = \dfrac{1}{\pi} \cdot \displaystyle\int^\pi_{-\pi} f(x) \sin kx \,dx \\
 +
c_k = \dfrac{1}{2\pi} \cdot \displaystyle\int^\pi_{-\pi} f(x) \exp(-ikx) \,dx </tex>
 +
}}
 +
 
=== Ядро Дирихле, ядро Фейера ===
 
=== Ядро Дирихле, ядро Фейера ===
 
{{Определение
 
{{Определение
Строка 112: Строка 239:
 
<tex>\forall h \in D</tex> определена функция <tex>K_h(x)</tex>, удовлетворяющая свойствам:
 
<tex>\forall h \in D</tex> определена функция <tex>K_h(x)</tex>, удовлетворяющая свойствам:
 
* <tex>\forall h \in D \  K_h \in L^1[-\pi; \pi] \quad  \left(\int\limits_{-\pi}^\pi K_h(t) = 1\right)</tex>
 
* <tex>\forall h \in D \  K_h \in L^1[-\pi; \pi] \quad  \left(\int\limits_{-\pi}^\pi K_h(t) = 1\right)</tex>
* L-нормы <tex>K_h</tex> огр. в свк.: <tex>\exists M \, \forall h \in D \quad \int\limits_{-\pi}^{\pi} |K_h| \;dt \leqslant M</tex>
+
* L-нормы <tex>K_h</tex> огр. в совокупности: <tex>\exists M \, \forall h \in D \quad \int\limits_{-\pi}^{\pi} |K_h| \;dt \leqslant M</tex>
* <tex>\forall \delta > 0 \int\limits_{E\delta} |K_n| \xrightarrow[h \to x_0]{} 0</tex>
+
* <tex>\forall \delta > 0 \int\limits_{E\delta} |K_h| \xrightarrow[h \to x_0]{} 0</tex>
 
Тогда семейство <tex>K_h</tex> называется аппроксимативной единицей.
 
Тогда семейство <tex>K_h</tex> называется аппроксимативной единицей.
 
}}
 
}}
Строка 125: Строка 252:
  
 
=== Метод суммирования средними арифметическими ===
 
=== Метод суммирования средними арифметическими ===
 +
{{Определение
 +
|definition=<tex>\sum a_n = \lim\limits_{n \to \infty} \dfrac{1}{n+1} \cdot \sum\limits_{k=0}^n S_k</tex>
 +
}}
  
 
=== Измеримое множество на простой двумерной поверхности в R^3 ===
 
=== Измеримое множество на простой двумерной поверхности в R^3 ===
 
=== Мера Лебега на простой двумерной поверхности в R^3 ===
 
=== Мера Лебега на простой двумерной поверхности в R^3 ===
 +
{{Определение
 +
|definition=
 +
<tex>\varphi \colon \mathbb R^2 \to M \subset \mathbb R^3</tex>.<br>
 +
 +
Мера в <tex>M</tex> — взвешенный образ меры Лебега в <tex>\mathbb R^2</tex> с весом <tex>|\varphi'_u \times \varphi'_v|</tex>
 +
}}
 +
 
=== Поверхностный интеграл первого рода ===
 
=== Поверхностный интеграл первого рода ===
 
{{Определение
 
{{Определение
Строка 133: Строка 270:
 
}}
 
}}
  
=== Кусочно-гладкая поверхность в R^3 ===
+
=== Кусочно-гладкая поверхность в ℝ<sup>3</sup> ===
 
{{Определение
 
{{Определение
 
|definition=<tex>M \subset \mathbb R^3</tex> называется кусочно-гладкой, если <tex>M</tex> представляет собой объединение:
 
|definition=<tex>M \subset \mathbb R^3</tex> называется кусочно-гладкой, если <tex>M</tex> представляет собой объединение:
Строка 140: Строка 277:
 
* конечного числа точек
 
* конечного числа точек
 
}}
 
}}
 
  
 
=== Сторона поверхности ===
 
=== Сторона поверхности ===
Строка 160: Строка 296:
  
 
=== Интеграл II рода ===
 
=== Интеграл II рода ===
 +
{{Определение
 +
|definition=
 +
<tex>
 +
\gamma \colon [a, b] \to \mathbb R^m, \quad V = (A_1, \dotsc, A_m) \\
 +
\displaystyle\int_\gamma A_1 \,dx_1 + \dotsb + A_m \,dx_m = \int_a^b \langle V, \gamma' \rangle \,dt
 +
</tex>
 +
}}
 +
 
=== Ориентация контура, согласованная со стороной поверхности ===
 
=== Ориентация контура, согласованная со стороной поверхности ===
 +
{{Определение
 +
|definition=Ориентация контура называется согласованной со стороной поверхности, если векторное произведение нормали и вектора скорости направлено внутрь контура.
 +
}}
 +
 
=== Ротор, дивергенция векторного поля ===
 
=== Ротор, дивергенция векторного поля ===
 
{{Определение
 
{{Определение
Строка 177: Строка 325:
 
{{Теорема
 
{{Теорема
 
|statement=
 
|statement=
<tex>(X, \mathfrak{A}, \mu) \quad U_n - </tex> измеримые функции на <tex>X, U_n(x) \geqslant 0 </tex> при всех <tex>x</tex><br>
+
<tex>(X, \mathfrak{A}, \mu) \quad U_n - </tex> измеримые функции на <tex>X, U_n(x) \geqslant 0 </tex> при почти всех <tex>x</tex>. Тогда
<tex>\int (\sum U_n(x)) d\mu = \sum (\int U_n(x) d\mu)</tex>
+
: <tex>\displaystyle\int\limits_X \Bigl(\displaystyle\sum U_n(x)\Bigr) d\mu = \displaystyle\sum \Bigl(\displaystyle\int\limits_X U_n(x) d\mu\Bigr)</tex>
 
|proof=
 
|proof=
 +
Пусть <tex>f_n(x) = U_1(x) + U_2(x) + \dotsb + U_n(x)</tex>, далее по т. Леви<br>
 +
<tex>f = \lim f_n</tex><br>
 +
<tex>0 \leqslant f_n \leqslant f_{n+1} \leqslant \dotsb</tex><br>
 +
Тогда выражение слева от знака равенства равно <tex>\displaystyle\int\limits_X f \,d\mu</tex>, а справа — <tex> \displaystyle\lim \int\limits_X \sum_{k=1}^n U_k(x) \,d\mu = \lim_{n \to +\infty}\Bigl(\int\limits_X f_n \,d\mu\Bigr) = \int\limits_X f \,d\mu</tex>
 
}}
 
}}
  
Строка 188: Строка 340:
 
<tex>\forall \epsilon > 0 \quad \exists \delta > 0 : \forall E \in \mathfrak{A} \quad \mu E < \delta \Rightarrow \int\limits_E |f|d\mu < \epsilon</tex>
 
<tex>\forall \epsilon > 0 \quad \exists \delta > 0 : \forall E \in \mathfrak{A} \quad \mu E < \delta \Rightarrow \int\limits_E |f|d\mu < \epsilon</tex>
 
|proof=
 
|proof=
 +
<tex>X_n = X (|f| > n) \quad X_n \supset X_{n+1} \supset ... \quad \bigcap X_n  = e</tex>, т.к. <tex>f</tex> - суммируема, <tex>\mu e = 0</tex><br>
 +
<tex>\nu E = \int\limits_E |f| d\mu</tex> - мера <tex>\nu</tex><br>
 +
<tex>\nu X < + \infty</tex> (т.к. <tex>f</tex> - суммируема и <tex>\int\limits_X |f| d\mu < +\infty</tex>)<br>
 +
Тогда по свойству непрерывности меры сверху: <tex>\nu X_n \to 0</tex><br>
 +
Запишем данное высказывание как <tex>\forall \epsilon > 0 \quad \exists n_\epsilon : \nu(X_{n_\epsilon}) < \dfrac{\epsilon}{2}</tex>, т.е. <tex>\int\limits_{X_{n_\epsilon}} |f| < \dfrac{\epsilon}{2}</tex><br>
 +
Теперь пусть <tex>\delta := \dfrac{\epsilon}{2 \cdot n_\epsilon}</tex><br>
 +
<tex>\int\limits_E |f| d\mu = \int\limits_{E \cap X_{n_\epsilon}} |f| d\mu + \int\limits_{E \cap X^C_{n_\epsilon}} |f| d\mu \leqslant \int\limits_{X_{n_\epsilon}} |f| d\mu + \int\limits_{E \cap X^C_{n_\epsilon}} n_\epsilon d\mu \leqslant \dfrac{\epsilon}{2} + n_\epsilon \cdot \mu E < \epsilon</tex>
 
}}
 
}}
  
Строка 197: Строка 356:
 
Тогда <tex>f_n, f</tex> - суммируемые и <tex>\int |f-f_n| d\mu \to 0</tex>
 
Тогда <tex>f_n, f</tex> - суммируемые и <tex>\int |f-f_n| d\mu \to 0</tex>
 
|proof=
 
|proof=
 +
<tex>f_n</tex> - суммируема, т.к. <tex>\int |f_n| \leqslant \int g < + \infty</tex><br>
 +
<tex>f</tex> - суммируема, т.к. <tex>\exists f_{n_k} \to f</tex> почти везде, <tex> |f_{n_k}| \leqslant g \Rightarrow |f| \leqslant g</tex><br>
 +
<tex>\int\limits_X |f_n - f| d\mu \to 0 ?</tex><br>
 +
Рассмотрим два случая:<br>
 +
1) <tex>\mu X < +\infty</tex><br>
 +
Берём <tex>\epsilon > 0 \quad X_n := X (|f_n - f| > \epsilon) \quad \mu X_n \to 0</tex><br>
 +
<tex>\int\limits_X |f_n - f| d\mu \leqslant \int\limits_{X_n} |f_n - f| d\mu + \int\limits_{X^C_n} |f_n - f| d\mu</tex><br>
 +
Для <tex>X_n</tex> выполнено <tex>|f_n - f| \leqslant |f_n| + |f| \leqslant 2 \cdot g</tex><br>
 +
А для <tex>X^C_n</tex> выполнено <tex> |f_n - f| < \epsilon</tex><br>
 +
Тогда <tex>\int\limits_{X_n} |f_n - f| d\mu + \int\limits_{X^C_n} |f_n - f| d\mu \leqslant \int\limits_{X_n} 2 \cdot g d\mu + \int\limits_{X^C_n} \epsilon d\mu \leqslant 2 \cdot \int\limits_{X_n} g + \epsilon \cdot \mu X \leqslant \epsilon \cdot (2 + \mu X)</tex><br>
 +
Получили <tex>\forall \epsilon > 0 \quad \exists N: \forall n > N \quad \int\limits_X |f_n - f| d\mu < \epsilon \cdot (2 + \mu X)</tex><br>
 +
Осталось найти номер <tex>N</tex>. Нужно взять такой, чтобы <tex>\mu X_n < \delta</tex>.<br>
 +
2) <tex>\mu X = +\infty</tex><br>
 +
TBD
 
}}
 
}}
  
Строка 202: Строка 375:
 
{{Теорема
 
{{Теорема
 
|statement=
 
|statement=
<tex>(X, \mathfrak{A}, \mu), f, f_n : X \rightarrow \tilde{\mathbb{R}}, f_n \rightarrow f </tex> почти везде <br>
+
<tex>(X, \mathfrak{A}, \mu), f, f_n \colon X \rightarrow \overline{\mathbb{R}}, f_n \rightarrow f </tex> почти везде <br>
 
<tex>\exists g</tex> - суммируемая и <tex>\forall n |f_n| \leqslant g</tex> для почти всех <tex>x</tex><br>
 
<tex>\exists g</tex> - суммируемая и <tex>\forall n |f_n| \leqslant g</tex> для почти всех <tex>x</tex><br>
Тогда <tex>f_n, f</tex> суммируемые и <tex>\int |f-f_n|d\mu \to 0</tex>
+
Тогда <tex>f_n, f</tex> суммируемые и <tex>\displaystyle\int |f-f_n|d\mu \to 0, \int_X f_n \to \int_X f</tex>
 
|proof=
 
|proof=
 +
Легко видеть, что <tex>f, f_n</tex> — суммируемые.<br>
 +
<tex>
 +
h_n := \sup(|f_n - f|, |f_{n+1} - f|, \dotsc) \\
 +
h_n \geqslant h_{n+1} \geqslant \dotsb; \qquad |f_n - f| \leqslant 2g \Rightarrow h_n \leqslant 2g
 +
</tex>
 +
 +
Кстати, <tex>\lim h_n = \varlimsup |f_n - f| = 0</tex> при п.в. <tex>x</tex>.
 +
 +
Рассмотрим ф-ии <tex>2g - h_n \geqslant 0</tex> — возр.
 +
: <tex>\lim \displaystyle\int_X (2g - h_n) = \int_X \lim(2g - h_n) = 2 \int_X g</tex>
 +
С другой стороны,
 +
: <tex>\lim \displaystyle\int_X (2g - h_n) = \lim\biggl(2 \int_X g - \int_X h_n\biggr) \Rightarrow \int_X h_n \to 0 \Rightarrow \int_X |f_n - f| \leqslant \int_X h_n</tex>
 
}}
 
}}
  
Строка 211: Строка 396:
 
{{Теорема
 
{{Теорема
 
|statement=
 
|statement=
<tex>(X, \mathfrak{A}, \mu), f_n \to f</tex> почти везде на <tex>X</tex>, и <tex>\exists C: \forall n \int {f_n d\mu} < C</tex><br>
+
<tex>(X, \mathfrak{A}, \mu), f_n \to f</tex> почти везде на <tex>X</tex>, и <tex>\exists C > 0: \forall n \displaystyle\int_X {f_n \;d\mu} \leqslant C</tex><br>
Тогда <tex>\int {f d\mu} < C</tex>
+
Тогда <tex>\displaystyle\int f \;d\mu \leqslant C</tex>
 
|proof=
 
|proof=
 +
<tex dpi=150>
 +
g_n = \inf(f_n, f_{n+1}, \dotsc)\\
 +
g_n(x) \leqslant g_{n+1}(x) \leqslant \dotsb \quad \lim g_n = \varliminf f_n = f\\
 +
\displaystyle\int g_n \leqslant \int f_n \leqslant C\\
 +
\int f \;d\mu = \lim_{n \to +\infty} \int g_n \leqslant C
 +
</tex>
 
}}
 
}}
  
Строка 219: Строка 410:
 
{{Теорема
 
{{Теорема
 
|statement=
 
|statement=
 +
<tex>f: X \times Y \rightarrow \mathbb{R}, \forall y \int\limits_X f(x, y) d\mu(x)</tex> - имеет смысл и выполнены 2 условия:<br>
 +
# <tex>f</tex> удовлетворяет условию <tex>L_{loc}(y_0)</tex>
 +
# <tex> y \rightarrow f(x, y)</tex> - непрерывна при всех <tex>x</tex> <br> <tex>f(x, y) \rightarrow f(x, y_0)</tex> при <tex>y \to y_0</tex> при всех <tex>x</tex> <br> Тогда <tex>I(y) = \int\limits_X f(x, y) d\mu(x)</tex> непрерывна в <tex>y_0</tex>
 
|proof=
 
|proof=
 +
Рассмотрим <tex>f_n(x) = f(x, y_n)</tex>, где <tex>y_n \rightarrow y_0, y_n \in (Y \cap U) \setminus \{a\}</tex>.
 +
Применим теорему Лебега для <tex>f_n</tex>.
 
}}
 
}}
 +
 
=== Правило Лейбница дифференцирования интеграла по параметру ===
 
=== Правило Лейбница дифференцирования интеграла по параметру ===
 
{{Теорема
 
{{Теорема
 
|statement=
 
|statement=
 +
<tex>f: X \times Y \rightarrow \mathbb{R}, Y \in \mathbb{R}</tex> - промежуток<br>
 +
 +
# <tex>\forall y \quad x \rightarrow f(x, y)</tex> - суммируема, <tex>I(y) = \int\limits_X f(x, y) d\mu(x)</tex>
 +
# <tex>\forall y</tex> при всех <tex>x \quad \exists^* f'_y(x, y)</tex>
 +
# <tex>y_0 \in Y \quad f'_y(x, y)</tex> удовлетворяет условию <tex>L_{loc}(y_0)</tex><br>Тогда <tex>I'(y_0) = \int\limits_X f'_y(x, y)d\mu(x)</tex>
 
|proof=
 
|proof=
 +
Пусть <tex>x \in X, y_0 + h \in Y, h \not = 0</tex><br>
 +
<tex>F(x, h) = \frac{f(x, y_0 + h) - f(x, y_0)}{h}</tex> <br>
 +
Т.к. <tex>\frac{I(y_0 + h) - I(y_0)}{h} = \int\limits_X \frac{f(x, y_0 + h) - f(x, y_0)}{h} d\mu(x) = \int\limits_X F(x, h) d\mu(x)</tex>, то при <tex>h \rightarrow 0</tex> сразу будет следовать теорема. Для доказательства законности этого перехода докажем, что <tex>F</tex> удовлетворяет <tex>L_{loc}</tex> в <tex>h = 0</tex>:
 +
 +
<tex>f'_y</tex> удовлетворяет условию <tex>L_{loc}</tex>, поэтому найдутся такие <tex>\delta</tex> и <tex>g</tex>, что <tex>|f'_y(x, y)| \leq g(x)</tex> при почти всех <tex>x</tex> и при <tex>y \in Y, 0 < |y - y_0| < \delta</tex>.
 +
 +
Теорема Лагранжа о среднем применённая к <tex>y \rightarrow f(x, y)</tex> на <tex>(y_0, y_0 + h)</tex> даст <tex>F(x, h) = f'_y(x, y_0 + \theta h)</tex>. Поэтому <tex>F(x, h) \leq g(x)</tex>.
 
}}
 
}}
 +
 
=== Вычисление интеграла Дирихле ===
 
=== Вычисление интеграла Дирихле ===
 
{{Теорема
 
{{Теорема
 
|statement=
 
|statement=
<tex>\int\limits_0^{+\infty} \dfrac{\sin \alpha x}{x}  = \dfrac{\pi}{2} \times sgn(\alpha)</tex>
+
<tex>\displaystyle\int\limits_0^{+\infty} \dfrac{\sin \alpha x}{x}  = \dfrac{\pi}{2} \cdot \operatorname{sgn}(\alpha)</tex>
 
|proof=
 
|proof=
 +
Можно, например, [[wikipedia:Dirichlet integral#Via the Dirichlet kernel|вот так]].
 
}}
 
}}
  
Строка 236: Строка 447:
 
{{Теорема
 
{{Теорема
 
|statement=
 
|statement=
 +
<tex> (X, \mathfrak{A}, \mu), (Y, B, ???)</tex><br>
 +
<tex> w \geqslant 0 </tex> - измеримая на <tex>X</tex> функция<br>
 +
<tex> \phi: X \rightarrow Y \quad \phi^{-1}(B) \in A</tex><br>
 +
<tex>v(B) = \displaystyle\int\limits_{\phi^{-1}(B)} w(x) d\mu</tex> - взвешенный образ <tex>\mu</tex> при отображении <tex>\phi, w </tex> - вес<br>
 +
Тогда: <tex>\forall Y_0 \in Y \displaystyle\int\limits_{Y_0} f(y) dv = \int\limits_{\phi^{-1}(Y_0)} f(\phi(x)) \cdot w(x) d\mu(x)</tex>
 
|proof=
 
|proof=
 +
Это очевидно верно, если <tex>f -</tex> характеристическая функция. По линейности интеграла это также верно и для простой неотрицательной <tex>f</tex>.
 +
 +
Для произвольной неотрицательной <tex>f</tex> рассмотрим последовательность простых неотрицательных функций <tex>f_n</tex> и по теореме Леви (предельный переход) теорем доказана для неотрицательных <tex>f</tex>.
 +
 +
Для отрицательных там надо что-то ещё сделать))))
 
}}
 
}}
 +
 
=== Критерий плотности ===
 
=== Критерий плотности ===
 
{{Теорема
 
{{Теорема
 
|statement=
 
|statement=
 +
<tex>(X, \mathfrak(A), \mu) \quad v, w</tex> - измеримые, <tex>w \geqslant 0</tex><br>
 +
<tex>w </tex> - плотность <tex>v</tex> относительно <tex>\mu \Leftrightarrow \forall T \in A \quad \mu(T) \times \inf(w) \leqslant v(T) \leqslant \mu(T) \times \sup(w)</tex>
 
|proof=
 
|proof=
 +
<tex>\Rightarrow)</tex> Очевидно<br>
 +
<tex>\Leftarrow)</tex> Пусть <tex>w > 0</tex> (без потери общности)<br>
 +
<tex>A = \bigcup\limits_{k \in \mathbb{Z}} A_k (q^k \leqslant w \leqslant q^{k-1}) \quad q \in (0, 1)</tex><br>
 +
<tex>q^k \cdot \mu A_k \leqslant \nu (A_k) \leqslant q^{k-1} \cdot \mu A_k</tex><br>
 +
<tex>q^k \cdot \mu A_k \leqslant \int\limits_{A_k} w d\mu \leqslant q^{k-1} \cdot \mu A_k</tex><br>
 +
<tex>q \cdot \int\limits_{A_k} w d\mu \leqslant q^k \cdot \mu A_k \leqslant \nu(A_k) \leqslant \dfrac{1}{q} \cdot q^k \cdot \mu(A_k) \leqslant \dfrac{1}{q} \cdot \int\limits_{A_k} w d\mu</tex><br>
 +
<tex>q \cdot \int\limits_{A_k} w d\mu \leqslant \nu(A_k) \leqslant \dfrac{1}{q} \cdot \int\limits_{A_k} w d\mu</tex><br>
 +
<tex>q \to 1-0</tex><br>
 +
<tex>\nu(A) = \int\limits_{A} w d\mu</tex>
 
}}
 
}}
 +
 
=== Лемма о множествах вполне положительности заряда ===
 
=== Лемма о множествах вполне положительности заряда ===
 
{{Теорема
 
{{Теорема
 
|statement=
 
|statement=
 +
<tex>(X, \mathfrak{A}, \mu), \quad A \in \mathfrak A, \quad \mu A \geqslant 0</tex><br>
 +
Тогда <tex>\exists B \subset A</tex> — множество положительности: <tex>\mu(B) \geqslant \mu(A)</tex>
 
|proof=
 
|proof=
 +
<div style="margin-left: 1em">
 +
{{Определение
 +
  |definition=<tex>C</tex> — мн-во <tex>\varepsilon</tex>-положительности,если <tex>\forall B \subset C \quad \mu B \geqslant -\varepsilon</tex>
 
}}
 
}}
=== Теорема Радона--Никодима ===
+
{{Утверждение
 +
  |statement= <tex>\forall \varepsilon > 0 \ A</tex> содержит мн-во <tex>\varepsilon</tex>-положительности.
 +
  |proof=
 +
#<tex>A</tex> — мн-во <tex>\varepsilon</tex>-положительности — очевидно
 +
#<tex>A</tex> не явл. мн-вом <tex>\varepsilon</tex>-положительности: <tex>\exists B_1 \subset A : \mu B_1 < -\varepsilon</tex><br><!--
 +
--><tex>C_1 := A \setminus B_1 \Rightarrow \mu C_1 > \mu A</tex>
 +
##<tex>C_1</tex> — мн-во <tex>\varepsilon</tex>-положительности — ОК
 +
##Иначе <tex>\exists B_2 \subset C_1 : \mu B_2 < -\varepsilon \quad C_2 := C_1 \setminus B_2 \quad \mu C_2 > \mu C_1</tex>
 +
#Продолжаем в том же духе — и рано или поздно приходим к успеху, т.к. иначе <tex>\mu \left( \bigcup B_i \right) = -\infty</tex>
 +
}}</div>
 +
 
 +
<tex>C_1 \subset A</tex> — мн-во 1-положительности: &nbsp; <tex>\mu C_1 \geqslant \mu A</tex><br>
 +
<tex>C_2 \subset C_1</tex> — мн-во <tex>1/2</tex>-положительности: &nbsp; <tex>\mu C_2 \geqslant \mu C_1</tex><br>
 +
<tex>\vdots</tex><br>
 +
<tex>C_n \subset C_{n-1}</tex> — мн-во <tex>1/n</tex>-положительности: &nbsp; <tex>\mu C_n \geqslant \mu C_{n-1}</tex><br>
 +
Пусть <tex>B = \bigcap C_i</tex><br>
 +
<tex>\mu B = \lim\limits_{i \to +\infty} \mu C_i \geqslant \mu A</tex>
 +
}}
 +
 
 +
=== Теорема Радона Никодима ===
 
{{Теорема
 
{{Теорема
 
|author=Радон, Никодим
 
|author=Радон, Никодим
Строка 255: Строка 513:
 
<tex>f</tex> — плотность <tex>\nu</tex> относительно <tex>\mu</tex>.
 
<tex>f</tex> — плотность <tex>\nu</tex> относительно <tex>\mu</tex>.
 
|proof=
 
|proof=
 +
==== Единственность ====
 +
<div style="margin-left: 1em">
 
   {{Лемма
 
   {{Лемма
   |statement=<tex>f, g</tex> — сумм. отн. <tex>\mu</tex>.
+
   |statement=Если <tex>f, g</tex> — сумм. отн. <tex>\mu</tex> и <tex>\displaystyle \forall A \in \mathfrak{A} \int_A f \, d\mu = \int_A g \, d\mu</tex>, то <tex>f = g</tex> п.в.
<tex>\forall A \in \mathfrak{A} \int_A f \, d\mu = \int_A g \, d\mu</tex>
+
  |proof=
 +
<tex>h := f - g</tex>.
 +
 
 +
<tex>
 +
\forall A \in \mathfrak A \quad \displaystyle\int_A h \,d\mu = 0 \\ \\
 +
X = X(h \geqslant 0) \cup X(h < 0) \\ \\
 +
\int\limits_{h \geqslant 0} h \,d\mu = 0, \quad \int\limits_{h < 0} h \,d\mu = 0
 +
</tex><br>
 +
Легко видеть, что <tex>\displaystyle\int_X |h| \,d\mu = 0 \ \Rightarrow h = 0</tex>&nbsp;п.в.
 
   }}
 
   }}
Хз если честно((99
+
</div>
 +
<h4>Существование</h4> <!-- мда чёт ==== не работают нифига((99 -->
 +
TBD
 
}}
 
}}
  
=== Теорема Радона--Никодима. Доказательство существования ===
 
{{Теорема
 
|statement=
 
|proof=
 
}}
 
 
=== Лемма об оценке мер образов кубов из окрестности точки дифференцируемости ===
 
=== Лемма об оценке мер образов кубов из окрестности точки дифференцируемости ===
 
{{Теорема
 
{{Теорема
 
|statement=
 
|statement=
 +
<tex>\phi: O \in \mathbb{R}^m \to \mathbb{R}^m, \quad a \in \mathbb{R}^m, f</tex> - диффиренцируема в <tex>a</tex><br>
 +
Пусть <tex>c > |\det \varphi'(a)| > 0, \quad \mu</tex> - мера Лебега на <tex>\mathbb{R}^m</tex><br>
 +
Тогда <tex>\exists U(a) \quad \forall</tex> куба <tex>Q \subset U(A), a \in Q</tex><br>
 +
<tex>\mu(\phi(Q))<c \cdot \mu(Q)</tex>
 
|proof=
 
|proof=
 
}}
 
}}
 +
 
=== Теорема о преобразовании меры при диффеоморфизме ===
 
=== Теорема о преобразовании меры при диффеоморфизме ===
 
{{Теорема
 
{{Теорема
 
|statement=
 
|statement=
 +
<tex>\phi \colon O \in \mathbb{R}^m \to \mathbb{R}^m</tex> - диффеоморфизм<br>
 +
Тогда <tex>\forall x \in \mathbb{R}^m \mu(\phi(a)) = \int\limits_a |\det \phi'(x)| \cdot d\mu(x)</tex>
 
|proof=
 
|proof=
 
}}
 
}}
 +
 
=== Теорема о гладкой замене переменной в интеграле Лебега ===
 
=== Теорема о гладкой замене переменной в интеграле Лебега ===
 
{{Теорема
 
{{Теорема
 
|statement=
 
|statement=
 +
<tex>\varphi\colon O \subset \mathbb{R}^m \to \mathbb{R}^m </tex> — диффеоморфизм<br>
 +
Пусть <tex>O_1 := \varphi(O), \quad f \geqslant 0 </tex> — измерима на <tex>O_1</tex><br>
 +
Тогда <tex>\int\limits_{O_1} f(y) d\mu = \int\limits_{O} (f * \varphi)(x) \cdot |\det \varphi'(x)| d\mu(x)</tex>
 
|proof=
 
|proof=
 
}}
 
}}
 +
 
=== Теорема о произведении мер ===
 
=== Теорема о произведении мер ===
 
{{Теорема
 
{{Теорема
 
|statement=
 
|statement=
 +
<tex>\mathbb{R}^n \Rightarrow \lambda_a \cdot \lambda_b = \lambda_{a+b}</tex>
 
|proof=
 
|proof=
 
}}
 
}}
 +
 
=== Принцип Кавальери ===
 
=== Принцип Кавальери ===
 
{{Теорема
 
{{Теорема
 
|statement=
 
|statement=
 +
<tex>(X, \mathfrak{A}, \mu), \quad (Y, \mathfrak{B}, \nu) \quad \mu, \nu</tex> - сигма конечные, полные; <tex>m = \mu \times \nu</tex><br>
 +
<tex>C</tex> измеримо в <tex>\mathfrak{A} \times \mathfrak{B}</tex><br>
 +
Тогда:
 +
 +
# <tex>C_x - \mu</tex> — измерима при всех <tex>x</tex>
 +
# <tex>x \mapsto \nu(x)</tex> измерима при всех <tex>x</tex>
 +
# <tex>mc = \int\limits_X \nu(C_x)d\mu(x)</tex>
 +
 +
Аналогично для <tex>C_y</tex>
 
|proof=
 
|proof=
 
}}
 
}}
 +
 
=== Теорема Тонелли ===
 
=== Теорема Тонелли ===
 
{{Теорема
 
{{Теорема
 
|statement=
 
|statement=
 +
<tex>(X, \mathfrak{A}, \mu), \quad (Y, \mathfrak{B}, v) \quad \mu v</tex> - сигма конечные, полные; <tex>m = \mu * v</tex><br>
 +
<tex>f: X \times Y \to \mathbb{R}, f \geqslant 0</tex> измеримая, <tex>f_x := y \to f(x, y)</tex><br>
 +
Тогда:
 +
 +
# <tex>f_x - v</tex>-измерима при почти всех <tex>x</tex>
 +
# <tex>f_y - \mu</tex>-измерима при почти всех <tex>y</tex>
 +
# <tex>x \to \phi(x) := \int f_x dv</tex> - <tex> \mu</tex>-измеримая функция
 +
# <tex>\int\limits_{X \times Y} f dm = \int \limits_X \phi(x) d\mu(x) = \int (\int f(x, y) dv(y)) d\mu(x) = \int (\int f(x, y) d\mu(x)) dv(y)</tex>
 
|proof=
 
|proof=
 
}}
 
}}
 +
 
=== Формула для Бета-функции ===
 
=== Формула для Бета-функции ===
 
{{Теорема
 
{{Теорема
 
|statement=
 
|statement=
 +
<tex>B(x, y) = \dfrac{\Gamma(x) \cdot \Gamma(y)}{\Gamma(x + y)}</tex>
 
|proof=
 
|proof=
 +
Вычислим интеграл <tex>I(u, v) = \displaystyle\iint\limits_{x, y > 0} e^{-(x^2 + y^2)} x^{2u - 1} y^{2v-1} \;dx dy</tex>
 +
 +
С одной стороны, <tex>I(u, v) = I(u) \cdot I(v)</tex>, где
 +
: <tex dpi=150>I(u) = \displaystyle\int\limits_0^{+\infty} e^{-x^2} x^{2u-1} \;dx = \dfrac12 \int\limits_0^{+\infty} e^{-t} t^{u-1} \;dt = \dfrac12 \Gamma(u)</tex>
 +
 +
С другой стороны, переходя к полярным координатам, получим:
 +
:<tex dpi=150>I(u, v) = \displaystyle\int\limits_0^{+\infty} e^{-r^2} r^{2u + 2v - 1} \;dr \int\limits_0^{\pi/2} \cos^{2u-1} \varphi \sin^{2v-1} \varphi \;d\varphi = {}\\
 +
{} = \dfrac12 \Gamma(u + v) \int\limits_0^{\pi/2} \cos^{2u-1} \varphi \sin^{2v-1} \varphi \;d\varphi</tex>
 +
 +
Сделаем замену <tex>\cos^2 \varphi = t</tex>:
 +
:<tex dpi=150>\displaystyle\int\limits_0^{\pi/2} \cos^{2u-1} \varphi \sin^{2v-1} \varphi \;d\varphi = \frac{1}{2} B(u, v)</tex>
 +
 +
Составляя два выражения для <tex>I(u, v)</tex>, получим <tex>B(x, y) = \dfrac{\Gamma(x) \cdot \Gamma(y)}{\Gamma(x + y)}</tex>
 
}}
 
}}
 +
 
=== Теорема Фубини ===
 
=== Теорема Фубини ===
 
{{Теорема
 
{{Теорема
 
|statement=
 
|statement=
 +
<tex>(X, \mathfrak{A}, \mu), \quad (Y, \mathfrak{B}, \nu) \quad \mu, \nu</tex> — сигма-конечные, полные; <tex>m = \mu \times \nu</tex><br>
 +
<tex>f \colon X \times Y \to \overline{\mathbb{R}}</tex> — <tex>m</tex>-сумм. Тогда:
 +
# <tex>C_x</tex> — суммируема при всех <tex>x</tex>
 +
# <tex> x \mapsto q(x) = \int f_x \,d\nu</tex> сумм при всех <tex>x</tex>
 +
# <tex>\int f \,d\nu = \int q \,d\mu</tex>
 +
 +
Аналогично для <tex>C_y</tex>
 
|proof=
 
|proof=
 +
<tex>f = f_+ - f_- \quad \int\limits_{X \times Y} f_\pm \,dm</tex> — кон.<br>
 +
<tex>\displaystyle\int_{X \times Y} f \,dm = \int_{X \times Y} f_+ \,dm - \int_{X \times Y} f_- \,dm</tex><br>
 +
<tex>\displaystyle\int(f_x)_+ , \int(f_x)_-</tex> — кон. при п.в. <tex>x</tex><br>
 +
Т.к. <tex>f_+ \geqslant 0 \Rightarrow \displaystyle\int_X \left( \int_Y (f_x)_+ \,d\nu \right) d\mu</tex> — кон. <tex> \Rightarrow \displaystyle\int_Y (f_x)_+\,d\nu</tex> — кон. при п.в. <tex>x</tex>
 +
 +
<tex>
 +
\varphi(x)_+ = \displaystyle\int_Y (f_x)_+ \,d\nu \\
 +
\varphi(x) = \displaystyle\int_Y (f_x)_+ \,d\nu - \displaystyle\int_Y (f_x)_- \,d\nu \\
 +
\int_X |\varphi(x)| \,d\mu = {} \\
 +
{} = \int_X \left| \int_Y (f_x)_+ - \int_Y (f_x)_- \right| \,d\mu \leqslant \int_X \left( \left| \int_Y (f_x)_+ \right| - \left|\int_Y (f_x)_-\right| \right) \,d\mu \\
 +
\int\limits_{X \times Y} f \,dm = \left(\int\limits_{X \times Y} f_+ \right) - \left(\int\limits_{X \times Y} f_- \right) = \int\limits_X \int\limits_Y f_+ - \int\limits_X \int\limits_Y f_- = {} \\
 +
{} = \int\limits_X \left(\int\limits_Y f_+ - \int\limits_Y f_- \right) = \int\limits_X \int\limits_Y f
 +
</tex>
 
}}
 
}}
=== Объем шара в $\mathbb R^m$ ===
+
 
 +
=== Объем шара в R^m ===
 
{{Теорема
 
{{Теорема
 
|statement=
 
|statement=
 +
<tex>V(B(0, r)) = \alpha \cdot r^n</tex><br>
 +
<tex>\alpha = \dfrac{(\sqrt{\pi})^n}{\Gamma\left(\dfrac{n}{2} + 1\right)}</tex>
 
|proof=
 
|proof=
 
}}
 
}}
=== Теорема о вычислении интеграла по мере Бореля--Стилтьеса (с леммой) ===
+
 
 +
=== Теорема о вычислении интеграла по мере Бореля — Стилтьеса (с леммой) ===
 +
{{Лемма
 +
|statement=
 +
<tex>(X, \mathfrak{A}, \mu), \quad h</tex> — измерима, почти везде конечна<br>
 +
<tex>H</tex> — функция распределения: <tex>H(t) = \mu X (h < t)</tex><br>
 +
<tex>\nu = h(\mu)</tex>, т.е. <tex>\nu(A) = \mu(h^{-1}(A))</tex><br>
 +
<tex>\mu_H</tex> — мера Бореля-Стилтьеса от <tex>H</tex><br>
 +
Тогда <tex>\mu_H \equiv \nu</tex> на <tex>B</tex> (Борелевской сигма-алгебре)
 +
|proof=
 +
<tex>[a, b) \quad \mu_H [a;\, b) = H(b-0) - H(a-0) = (*)</tex><br>
 +
<tex>H\left(b - \dfrac1n\right) = \mu X\left(h < b - \dfrac1n\right)</tex><br>
 +
<tex>H(b-0) = \lim\limits_{n \to +\infty} \mu X \left(h < b - \dfrac1n\right) = \mu X(h<b)</tex> &nbsp; <tex>\left(\displaystyle \bigcup X \left(h < b - \dfrac1n\right) = X(h<b)\right)</tex><br>
 +
<tex>(*) = H(b) - H(a) = \mu X(h < b) -\mu X(h < a) = \mu X(a \leqslant h < b) = \mu h^{-1} [a, b)</tex>&nbsp;<tex>{ } = \nu [a, b)</tex>
 +
}}
 +
 
 +
 
 
{{Теорема
 
{{Теорема
 
|statement=
 
|statement=
|proof=
+
<tex>f: \mathbb{R} \to \mathbb{R}, \quad f \geqslant 0</tex> измерима относительно <tex>B</tex><br>
 +
Остальное из прошлой леммы<br>
 +
Тогда: <tex>\int\limits_X f(h(x)) d\mu(x) = \int\limits_R f(t) d\mu_h(t)</tex>
 +
|proof=Ну тут тип просто замена в интеграле)))
 
}}
 
}}
 +
 
=== Теорема о вложении пространств L^p ===
 
=== Теорема о вложении пространств L^p ===
 
{{Теорема
 
{{Теорема
Строка 324: Строка 680:
  
 
# <tex dpi=150>1 \leqslant s < r < +\infty</tex>, тогда <tex dpi=150>L^r \subset L^s</tex>
 
# <tex dpi=150>1 \leqslant s < r < +\infty</tex>, тогда <tex dpi=150>L^r \subset L^s</tex>
# <tex dpi=150>\| f \|_s \leqslant (\mu(X))^{\frac{1}{s} - \frac{1}{r}} \times \| f \|_r</tex>
+
# <tex dpi=150>\| f \|_s \leqslant (\mu(X))^{\frac{1}{s} - \frac{1}{r}} \cdot \| f \|_r</tex>
 
|proof=
 
|proof=
1. Напрямую следует из 2
+
# Напрямую следует из 2
 +
# Пусть<br><!--
 +
--><tex dpi=150> \dfrac{r}{s} = p > 1</tex><br><!--
 +
--><tex dpi=150> q = \dfrac{r}{r - s}</tex><p><!--
 +
-->Тогда: <tex dpi=150>\| f \|^s_s = \int\limits_X |f|^s = \int\limits_X |f|^s \cdot 1 \leqslant \left(\int\limits_X |f|^{s \cdot \frac{r}{s}}\right)^\frac{s}{r} \cdot \left(\int\limits_X 1^{\frac{r}{r-s}}\right)^\frac{r-s}{r} = \| f \|_r^s \cdot (\mu(X))^{1-\frac{s}{r}}</tex> (по Гёльдеру)</p>
 +
}}
  
2. Пусть
+
=== Полнота L^p ===
 +
{{Теорема
 +
|statement=<tex>(X, \mathfrak{A}, \mu), L^p(X)</tex> — полное <tex>(1 \leqslant p < +\infty)</tex>
 +
|proof=
 +
<tex>f_n</tex> — фундамтельная в <tex>L^p</tex><br>
 +
Строим кандидата на роль предела:<br>
 +
<tex dpi=150>\varepsilon := \dfrac{1}{2} \quad \exists N_1 \quad \forall m, n \geqslant N_1 \quad \|f_m - f_n\|_p < \dfrac{1}{2}\\ \\
 +
    \varepsilon := \dfrac{1}{4} \quad \exists N_2 > N_1 \quad \forall m, n \geqslant N_2 \quad \|f_m - f_n\|_p < \dfrac{1}{4}\\ \\
 +
    \varepsilon := \dfrac{1}{8} \quad \dots</tex><br>
  
<tex dpi=150> \dfrac{r}{s} = p > 1</tex>
+
Очевидно, что <tex>\sum\limits_{k=1}^{+\infty} \|f_{N_{k+1}} - f_{N_k}\| < 1</tex><br>
  
<tex dpi=150> q = \dfrac{r}{r - s}</tex>
+
Рассмотрим <tex>S(x) = \sum\limits_{k=1}^{+\infty} |f_{N_{k+1}}(x) - f_{N_k}(x)| \in [0; +\infty]</tex><br>
  
Тогда: <tex dpi=150>\| f \|^s_s = \int\limits_X |f|^s = \int\limits_X |f|^s \cdot 1 \leqslant (\int\limits_X |f|^{s \cdot \frac{r}{s}})^\frac{s}{r} \times (\int\limits_X 1^{\frac{r}{r-s}})^\frac{r-s}{r} = \| f \|_r^s \times (\mu(X))^{1-\frac{s}{r}}</tex> (По Гельдеру)
+
<tex>\|S_N\|_p = \|\sum ... \|_p \leqslant \sum\limits_{k=1}^{N} \|f_{N_{k+1}} - f_{N_k}\|_p < 1</tex><br>
}}
 
  
=== Полнота L^p ===
+
Т.е. <tex>\displaystyle\int\limits_X |S_N(x)|^p d\mu(x) < 1</tex><br>
{{Теорема
+
При всех <tex>x \quad S_N(x) \to S(x)</tex><br>
|statement=<tex>(X, \mathfrak{A}, \mu), L^p(X)</tex> - полное <tex>(1 \leqslant p < +\infty)</tex>
+
<br>
|proof=Ну там сложно что-то((((
+
По теореме Фату <tex>\displaystyle\int\limits_X |S(x)|^p < 1</tex>, т.е. <tex>|S(x)|^p</tex> - суммируема<br>
 +
Значит <tex>|S(x)|</tex> почти везде конечна. <tex> \Rightarrow </tex> Ряд <tex> \sum f_{N_{k+1}}(x) - f_{N_k}(x)</tex> абсолютно сходится при почти всех <tex>x</tex>.<br>
 +
<br>
 +
<tex>f(x) = f_{N_1}(x) + \sum f_{N_{k+1}}(x) - f_{N_k}(x)</tex><br>
 +
При всех <tex>x \quad f(x) = \lim\limits_{k \to +\infty} f_{N_1} + \sum\limits_{i=1}^{k}(...) = \lim\limits_{k \to +\infty} f_{N_{k+1}}(x)</tex><br>
 +
<tex>\|f\|_p \leqslant \|f_{N_1}\|_p + \sum \|f_{N_{k+1}} - f_{N_k}\|_p</tex> — конечна<br>
 +
<tex>\|f(x)-f_n(x)\|_p \to 0 ?</tex><br>
 +
<br>
 +
<tex>\forall \varepsilon > 0 \quad \exists N \quad \forall m, n > N \quad \|f_n-f_m\|_p^p < \varepsilon^p</tex><br>
 +
Возьмём <tex>m:=N_k > N</tex><br>
 +
<tex>\|f_n-f_{N_k}\|_p^p < \epsilon^p</tex><br>
 +
<tex>\displaystyle\int\limits_X |f_n(x) - f_{N_k}(x)|^p d\mu(x) < \varepsilon^p</tex><br>
 +
<br>
 +
По теореме Фату:<br>
 +
<tex>\displaystyle\int\limits_X |f_n(x) - f(x)|^p d\mu < \varepsilon^p \Rightarrow f_n \rightrightarrows f</tex>
 
}}
 
}}
  
Строка 346: Строка 728:
 
{{Теорема
 
{{Теорема
 
|statement=
 
|statement=
<tex>(X, \mathfrak{A}, \mu), f - </tex> ступенчатая <tex> = \sum_{k=1}^{n} C_k \times</tex> <tex dpi=160>\chi_{Ek}</tex>
+
<tex>(X, \mathfrak{A}, \mu), f - </tex> ступенчатая &nbsp;<tex dpi=160>{} = \sum\limits_{k=1}^{n} C_k \times \chi_{E_k}</tex>
  
 
<tex>X = \bigsqcup X_k</tex>
 
<tex>X = \bigsqcup X_k</tex>
Строка 354: Строка 736:
 
в <tex>L^p(X, \mu) (1 \leqslant p \leqslant +\infty)</tex> множество ступенчатых функций плотно
 
в <tex>L^p(X, \mu) (1 \leqslant p \leqslant +\infty)</tex> множество ступенчатых функций плотно
 
|proof=
 
|proof=
 +
#  <tex>p = \infty \quad f \in L^\infty \quad \|f\|_\infty = \operatorname{ess\,sup} |f| < +\infty</tex><br><!--
 +
-->Поправив <tex>f</tex> на множестве нулевой меры, получим <tex>\forall x \in X \ |f(x)| \leqslant \|f\|_\infty</tex><br><!--
 +
--><tex>f</tex> — изм. огр., <tex>\exists h_n : \sup |f - h_n| \to 0 \Rightarrow \|f - h_n\|_\infty = \operatorname{ess\,sup} |f - h_n| \leqslant \sup |f - h_n|</tex>
 +
#  <tex>p < +\infty \quad f \in L^p \quad B(f, \varepsilon)</tex> — есть ли здесь ступ. ф-ия?<br><!--
 +
--><tex>f \geqslant 0 \quad \exists</tex> ступ. <tex>h_n : h_n \leqslant h_{n+1} \leqslant \dots \quad h_n \to f, h_n \leqslant f</tex><br><!--
 +
--><tex>\displaystyle\int\limits_X |f - h_n| \to 0</tex><br><!--
 +
--><tex>\|f - h_n\|^p_p = \displaystyle\int\limits_X |f - h_n|^p d\mu(x) \xrightarrow[n \to +\infty]{} 0</tex> (по т. Лебега).
 
}}
 
}}
  
Строка 359: Строка 748:
 
{{Теорема
 
{{Теорема
 
|statement=
 
|statement=
<tex>F_1, F_2 - </tex> два непересекающихся замкнутых множества из <tex>\mathbb{R}^m</tex><br>
+
<tex>F_0, F_1 - </tex> два непересекающихся замкнутых множества из <tex>\mathbb{R}^m</tex><br>
Тогда <tex>\exists f: \mathbb{R}^m \to \mathbb{R}</tex> (непрырывная)<tex>: f|_{F_1}=0, f|_{F_2}=1</tex>
+
Тогда <tex>\exists f: \mathbb{R}^m \to \mathbb{R}</tex> (непрырывная)<tex>: f|_{F_0}=0, f|_{F_1}=1</tex>
 
|proof=
 
|proof=
 +
<tex>\forall</tex> замкн. <tex>F</tex> и <tex>\forall</tex> откр. <tex>G \supset F</tex> <tex>\exists</tex> откр. <tex>H : F \subset H \subset \overline H \subset G</tex>.<br>
 +
<tex>\exists U(F_0), U(F_1)</tex> — откр.: <tex>U(F_0) \cap U(F_1) = \varnothing</tex><br>
 +
<tex>F_0 \subset G_0 \subset \overline{G_0} \subset F_1^c = G_1</tex><br>
 +
<tex>\overline{G_0} \subset G_1 \quad \exists G_{1/2} \quad \overline{G_0} \subset G_{1/2} \subset \overline{G_{1/2}} \subset G_1</tex><br>
 +
Аналогично можно ввести <tex>G_{1/4}, G_{3/4}</tex> и так далее <tex>G_{\alpha}</tex> для любого двоично-рационального <tex>\alpha \in [0; 1]</tex>.
 +
 +
<tex>f(x) := \sup \{x \in G_\alpha \mid \alpha</tex> — дв. рац. <tex> \}</tex> — непр.
 +
<tex>(a, b) \subset [0, 1], a</tex> — дв. рац.  &nbsp;&nbsp;<tex>{}\quad f^{-1}(a, b) = \!\!\!\!\!\!\displaystyle\bigcup_{\substack{\alpha \in (a, b) \\ \alpha \text{ is dyadic rat.}}}\!\!\!\!\!\! G_\alpha \setminus \overline{G_a}</tex>
 
}}
 
}}
  
Строка 375: Строка 772:
 
|statement=
 
|statement=
 
<tex>f_n(x) = f(x + h)</tex>
 
<tex>f_n(x) = f(x + h)</tex>
# <tex>f</tex> - равномерно непрерывна на <tex>\mathbb{R}^m \Rightarrow lim_{h \to 0} \| f_n - f \|_\infty = 0</tex>
+
# <tex>f</tex> - равномерно непрерывна на <tex>\mathbb{R}^m \Rightarrow \displaystyle\lim_{h \to 0} \| f_n - f \|_\infty = 0</tex>
# <tex>1 \leqslant p < +\infty \quad f \in L^p (\mathbb{R}^m) \Rightarrow lim_{h \to 0} \| f_n - f \|_p = 0</tex>
+
# <tex>1 \leqslant p < +\infty \quad f \in L^p (\mathbb{R}^m) \Rightarrow \displaystyle\lim_{h \to 0} \| f_n - f \|_p = 0</tex>
# <tex>f \in \tilde{C}[0, T] \Rightarrow lim_{h \to 0} \| f_n - f \|_\infty = 0</tex>
+
# <tex>f \in \widetilde{C}[0, T] \Rightarrow \displaystyle\lim_{h \to 0} \| f_n - f \|_\infty = 0</tex>
# <tex>1 \leqslant p < +\infty \quad f \in L^p[0, T] \Rightarrow lim_{h \to 0} \| f_n - f \|_p = 0</tex>
+
# <tex>1 \leqslant p < +\infty \quad f \in L^p[0, T] \Rightarrow \lim\limits_{h \to 0} \| f_n - f \|_p = 0</tex>
 
|proof=
 
|proof=
 
}}
 
}}
Строка 385: Строка 782:
 
{{Теорема
 
{{Теорема
 
|statement= Пусть есть ГП
 
|statement= Пусть есть ГП
# <tex>x_n \to x, y_n \to y \quad</tex> Тогда <tex><x_n, y_n> \to <x, y></tex>
+
# <tex>x_n \to x, y_n \to y \quad</tex> Тогда <tex>\langle x_n, y_n\rangle \to \langle x, y \rangle</tex>
# <tex>\sum_{n=1}^{+\infty} x_n - </tex> ряд, сходящийся в ГП. Тогда <tex>\forall y <y, \sum_{n=1}^{+\infty} x_n> = \sum_{n=1}^{+\infty} <y, x_n></tex>
+
# <tex>\displaystyle\sum_{n=1}^{+\infty} x_n - </tex> ряд, сходящийся в ГП. Тогда <tex>\forall y \ \bigl\langle y, \sum_{n=1}^{+\infty} x_n \bigr\rangle = \displaystyle\sum_{n=1}^{+\infty} \langle y, x_n \rangle</tex>
# <tex>\sum_{n=1}^{+\infty} x_n - </tex> ортогональный ряд. Тогда <tex>\sum_{n=1}^{+\infty} x_n - </tex> сходится <tex>\Leftrightarrow \sum_{n=1}^{+\infty} \| x_n \| - </tex> сходится.
+
# <tex>\displaystyle\sum_{n=1}^{+\infty} x_n - </tex> ортогональный ряд. Тогда <tex>\displaystyle\sum_{n=1}^{+\infty} x_n - </tex> сходится <tex>\Leftrightarrow \displaystyle\sum_{n=1}^{+\infty} \| x_n \| - </tex> сходится.
 
|proof=  
 
|proof=  
 
}}
 
}}
Строка 394: Строка 791:
 
{{Теорема
 
{{Теорема
 
|statement=
 
|statement=
<tex>\mathfrak{H} -</tex> ГП
+
<tex>\mathcal{H} -</tex> ГП
  
<tex>\{e_k\} - </tex> Ортогональная система. <tex> \quad x = \sum_{k=1}^{+\infty} C_k \times e_k</tex>
+
<tex>\{e_k\} - </tex> Ортогональная система. <tex>x = \displaystyle\sum_{k=1}^{+\infty} C_k \cdot e_k</tex>
  
 
Тогда:
 
Тогда:
  
 
# <tex>\{e_k\} - </tex> ЛНЗ
 
# <tex>\{e_k\} - </tex> ЛНЗ
# <tex>\dfrac{<x, e_k>}{\|e_k\|^2} = C_k</tex>
+
# <tex>\dfrac{\langle x, e_k \rangle}{\|e_k\|^2} = C_k</tex>
# <tex>C_k \times e_k - </tex> это проекция <tex>X</tex> на 1-номерное подпространство, порождённое <tex>e_k</tex>.<br> <tex> x = C_k \times e_k + z \Rightarrow z \perp e_k </tex>
+
# <tex>C_k \cdot e_k - </tex> это проекция <tex>X</tex> на 1-номерное подпространство, порождённое <tex>e_k</tex>.
 +
: <tex> x = C_k \cdot e_k + z \Rightarrow z \perp e_k </tex>
 
|proof=
 
|proof=
 
}}
 
}}
Строка 409: Строка 807:
 
{{Теорема
 
{{Теорема
 
|statement=
 
|statement=
<tex>\{e_k\} - </tex> Ортогональная система в <tex>\mathfrak{H}, x \in \mathfrak{H}</tex>
+
<tex>\{e_k\} - </tex> Ортогональная система в <tex>\mathcal{H}, x \in \mathcal{H}</tex>
  
<tex>S_n = \sum_{k=1}^{n} C_k (x) \times e_k - </tex> частичные суммы ряда Фурье
+
<tex>S_n = \displaystyle\sum_{k=1}^{n} C_k (x) \cdot e_k - </tex> частичные суммы ряда Фурье
  
<tex>\alpha_n := Lin(e_1, ..., e_n)</tex>
+
<tex>\alpha_n := \operatorname{Lin}(e_1, \dotsc, e_n)</tex>
  
 
Тогда:
 
Тогда:
Строка 422: Строка 820:
  
 
Следствие:
 
Следствие:
<tex>\sum |C_k(x)|^2 \times \| e_k \|^2 \leqslant \|x\|^2</tex>
+
<tex>\displaystyle\sum |C_k(x)|^2 \times \| e_k \|^2 \leqslant \|x\|^2</tex> (Неравенство Бесселя)
  
 
|proof=
 
|proof=
 
}}
 
}}
  
=== Теорема Рисса -- Фишера о сумме ряда Фурье. Равенство Парсеваля ===
+
=== Теорема Рисса Фишера о сумме ряда Фурье. Равенство Парсеваля ===
 
{{Теорема
 
{{Теорема
 
|statement=
 
|statement=
<tex>\{e_k\} - </tex> Ортогональная система в <tex> \mathfrak{H}, x \in  \mathfrak{H}</tex>
+
<tex>\{e_k\} - </tex> Ортогональная система в <tex>\mathcal{H}, x \in  \mathcal{H}</tex>
  
# Ряд Фурье <tex>x</tex> сходится в <tex>\mathfrak{H}</tex>
+
# Ряд Фурье <tex>x</tex> сходится в <tex>\mathcal{H}</tex>
# <tex>x = \sum_{k=1}^{+\infty} C_k(x) \times e_k + z, </tex> тогда <tex>\forall k \quad z \perp e_k</tex>
+
# <tex>x = \displaystyle\sum_{k=1}^{+\infty} C_k(x) \cdot e_k + z, </tex> тогда <tex>\forall k \quad z \perp e_k</tex>
# <tex>x = \sum_{k=1}^{+\infty} C_k(x) \times e_k \Leftrightarrow \sum_{k=1}^{+\infty} |C_k (x)|^2 \times \|e_k\|=\|x\|^2</tex> (Равенство Парсеваля)
+
# <tex>x = \displaystyle\sum_{k=1}^{+\infty} C_k(x) \times e_k \Leftrightarrow \displaystyle\sum_{k=1}^{+\infty} |C_k (x)|^2 \cdot \|e_k\|=\|x\|^2</tex> (Равенство Парсеваля)
 
|proof=
 
|proof=
 
}}
 
}}
Строка 454: Строка 852:
 
<tex>T(x) - </tex> тригонометрический ряд, <tex>\quad S_n(x) - </tex> частичные суммы
 
<tex>T(x) - </tex> тригонометрический ряд, <tex>\quad S_n(x) - </tex> частичные суммы
  
Пусть <tex>\exists f \in L^1[-\pi,\pi] \quad S_n \to f </tex> в пространстве <tex>L^1</tex>
+
Пусть <tex>f \in L^1[-\pi,\pi] \quad S_n \to f </tex> в пространстве <tex>L^1</tex>
  
 
Тогда:
 
Тогда:
  
# <tex>a_k = \dfrac{1}{\pi} \times \int_{-\pi}^{\pi} {f(x) \times \cos {kx} dx}</tex>
+
# <tex>a_k = \dfrac{1}{\pi} \cdot \displaystyle\int\limits_{-\pi}^{\pi} {f(x) \cdot \cos {kx} \;dx}</tex>
# <tex>b_k = \dfrac{1}{\pi} \times \int_{-\pi}^{\pi} {f(x) \times \sin {kx} dx}</tex>
+
# <tex>b_k = \dfrac{1}{\pi} \cdot \displaystyle\int\limits_{-\pi}^{\pi} {f(x) \cdot \sin {kx} \;dx}</tex>
# <tex>c_k = \dfrac{1}{2 \pi} \times \int_{-\pi}^{\pi} {f(x) \times e^{-ikx} dx}</tex>
+
# <tex>c_k = \dfrac{1}{2 \pi} \cdot \displaystyle\int\limits_{-\pi}^{\pi} {f(x) \cdot e^{-ikx} \;dx}</tex>
  
 
|proof=
 
|proof=
 
}}
 
}}
  
=== Теорема Римана--Лебега ===
+
=== Теорема Римана Лебега ===
 
{{Теорема
 
{{Теорема
 
|statement=
 
|statement=
<tex>E \in \mathbb{R} - </tex> измеримо, <tex>f \in L^1(E)</tex><br>
+
<tex>E \subset \mathbb{R}</tex> измеримо, <tex>f \in L^1(E)</tex><br>
  
Тогда <tex>\int\limits_E {f(x) \times e^{ikx} \times dx} \to_{k \to \infty} 0</tex> (То же самое можно и с <tex>\cos {x}</tex> и <tex>\sin {x}</tex> вместо <tex>e^{ikx}</tex>)
+
Тогда <tex>\displaystyle\int\limits_E {f(x) \cdot e^{ikx} \; dx} \xrightarrow[k \to \infty]{} 0</tex> (То же самое можно и с <tex>\cos {x}</tex> и <tex>\sin {x}</tex> вместо <tex>e^{ikx}</tex>)
 
|proof=
 
|proof=
 
}}
 
}}
Строка 481: Строка 879:
 
<tex>f(x) = g(x) </tex> при <tex> x \in (x_0 - \delta, x_0 + \delta)</tex><br>
 
<tex>f(x) = g(x) </tex> при <tex> x \in (x_0 - \delta, x_0 + \delta)</tex><br>
  
Тогда <tex>S_n(f, x_0) - S_n(g, x_0) \to_{n \to +\infty} 0</tex>
+
Тогда <tex>S_n(f, x_0) - S_n(g, x_0) \xrightarrow[n \to +\infty]{} 0</tex>
 
|proof=
 
|proof=
 
}}
 
}}
Строка 489: Строка 887:
 
|statement=
 
|statement=
 
<tex>f \in L^1[-\pi, \pi] \quad x_0 \in [-\pi, \pi] \quad S \in \mathbb{R}</tex><br>
 
<tex>f \in L^1[-\pi, \pi] \quad x_0 \in [-\pi, \pi] \quad S \in \mathbb{R}</tex><br>
Пусть <tex>\int\limits_0^\pi \dfrac{|f(x_0+t)+f(x_0-t)-2S|}{t} \times dt < +\infty </tex><br>
+
Пусть <tex>\displaystyle\int_0^\pi \dfrac{|f(x_0+t)+f(x_0-t)-2S|}{t} \; dt < +\infty </tex><br>
Тогда <tex>S_n(f, x_0) \to_{n \to +\infty} S</tex>
+
Тогда <tex>S_n(f, x_0) \xrightarrow[n \to +\infty]{} S</tex>
 
|proof=
 
|proof=
 
}}
 
}}
Строка 502: Строка 900:
 
{{Теорема
 
{{Теорема
 
|statement=
 
|statement=
<tex>f \in L^p \quad k \in L^q[-\pi, \pi] \quad (\dfrac{1}{p} + \dfrac{1}{q} = 1) \quad 1 \leqslant p < +\infty</tex><br>
+
<tex>f \in L^p \quad k \in L^q[-\pi, \pi] \quad \left(\dfrac{1}{p} + \dfrac{1}{q} = 1 \right) \quad 1 \leqslant p < +\infty</tex><br>
Тогда <tex>f \times k</tex> - непрерывна на <tex>[-\pi, \pi]</tex><br>
+
Тогда <tex>f * k</tex> - непрерывна на <tex>[-\pi, \pi]</tex><br>
<tex>\|f \times k \|_1 \leqslant \|f\|_p \times \|k\|_q</tex>
+
<tex>\|f * k \|_1 \leqslant \|f\|_p * \|k\|_q</tex>
 
|proof=
 
|proof=
 
}}
 
}}
Строка 511: Строка 909:
 
{{Теорема
 
{{Теорема
 
|statement=
 
|statement=
<tex>K_n - </tex> апроксимативная единица
+
<tex>K_n</tex> — аппроксимативная единица.
  
 
Тогда <tex>(h \to h_0)</tex>:
 
Тогда <tex>(h \to h_0)</tex>:
  
# <tex>f \in \tilde{C}[-\pi, \pi] \quad f \times K_n \rightrightarrows_{h \to h_0} f</tex>
+
# <tex>f \in \widetilde{C}[-\pi, \pi] \quad f * K_n \operatorname*{\rightrightarrows}\limits_{h \to h_0} f</tex>
# <tex>f \in L^p[-\pi, \pi] \quad \|f \times K_n - f \|_p \to 0, h \to 0</tex>
+
# <tex>f \in L^p[-\pi, \pi] \quad \|f * K_n - f \|_p \to 0, h \to 0</tex>
# <tex>f \in L^1, f - </tex> непр. <tex>x_0 \quad K_n - </tex> ??? а.е.<br> <tex>f \times K_n - </tex> непрерывна в окрестности <tex>x_0</tex> <br> <tex>(f \times K_n)(x_0) \to_{h \to h_0} f(x_0)</tex>
+
# <tex>f \in L^1, f</tex> непр. <tex>x_0 \quad K_n - </tex> ??? а.е.<br>
 +
<tex>f * K_n</tex> непрерывна в окрестности <tex>x_0</tex> <br>
 +
<tex>(f * K_n)(x_0) \xrightarrow[h \to h_0]{} f(x_0)</tex>
 
|proof=
 
|proof=
 
}}
 
}}
Строка 524: Строка 924:
 
{{Теорема
 
{{Теорема
 
|statement=
 
|statement=
 +
<tex>\sum a_n = S \Rightarrow \sum a_n </tex> (по методу средних арифметических) <tex> = S</tex>
 
|proof=
 
|proof=
 +
<tex dpi=150>\sum a_n </tex> (по методу средних арифметических) <tex> = \lim\limits_{n \rightarrow \infty} \frac 1{n + 1} \sum\limits_{k = 0}^n S_k</tex>
 +
<tex dpi=150>
 +
\left|\dfrac{\sum_{k=0}^n S_k}{n+1} - S\right| = \left|\sum\limits_{k=0}^n \dfrac{S_k-S}{n+1}\right| \leqslant \sum\limits_{k=0}^{n} \dfrac{|S_k-S|}{n+1}\\
 +
\forall \varepsilon > 0 \quad \exists N_1 \quad \forall n > N_1 \quad |S_n - S| < \dfrac{\varepsilon}{2} \\
 +
\sum\limits_{k=0}^{N_1} \dfrac{|S_k-S|}{n+1} + \sum\limits_{k=N_1 + 1}^{n} \dfrac{|S_k-S|}{n+1} < \varepsilon</tex>
 
}}
 
}}
 +
 
=== Теорема Фейера ===
 
=== Теорема Фейера ===
 
{{Теорема
 
{{Теорема
 
|statement=3 пункта:
 
|statement=3 пункта:
# <tex> f \in \tilde{C}[-\pi, \pi] \Rightarrow \sigma_n(f, x) \rightrightarrows_{n \to \infty} f(x)</tex>
+
# <tex> f \in \tilde{C}[-\pi, \pi] \Rightarrow \sigma_n(f, x) \operatorname*{\rightrightarrows}\limits_{n \to \infty} f(x)</tex>
# <tex> f \in L^p[-\pi, \pi] \Rightarrow \|\sigma_n(f, x) - f \|_p \to_{n \to \infty} 0</tex>
+
# <tex> f \in L^p[-\pi, \pi] \Rightarrow \|\sigma_n(f, x) - f \|_p \xrightarrow[n \to \infty]{} 0</tex>
# <tex> f \in L^1, f - </tex> непр. <tex> x \Rightarrow \sigma_n(f, x) \to_{n \to \infty} f(x)</tex>
+
# <tex> f \in L^1, f - </tex> непр. <tex> x \Rightarrow \sigma_n(f, x) \xrightarrow[n \to \infty]{} f(x)</tex>
 
|proof=
 
|proof=
 
}}
 
}}
Строка 544: Строка 951:
 
=== Формула Грина ===
 
=== Формула Грина ===
 
{{Теорема
 
{{Теорема
|statement=
+
|statement=<tex>\mathbb R^2</tex> — ориент. с помощью нумерации координат.
 +
<tex>D \subset \mathbb R^2</tex> — компактное, связное, односвязное, с <tex>C^2</tex>-гладкой границей.<br>
 +
<tex>(P, Q)</tex> — гладкое векторное поле.<br>
 +
Пусть граница <tex>D (\partial D)</tex> ориентирована согласованно с ориентацией плоскости.
 +
Тогда <tex>\displaystyle\int_{\partial D} P \,dx + Q \,dy = \displaystyle\iint_D \left(\dfrac{\partial Q}{\partial x} - \dfrac{\partial P}{\partial y}\right) dx\, dy</tex>
 
|proof=
 
|proof=
 
}}
 
}}
 +
 
=== Формула Стокса ===
 
=== Формула Стокса ===
 
{{Теорема
 
{{Теорема
|statement=
+
|statement=<tex>D \subset \mathbb R^3</tex> — простая гладкая поверхность в <tex>\mathbb R^3</tex>,
 +
<tex>\partial D</tex> — <tex>C^2</tex>-гладкая кривая,<br>
 +
<tex>n_0</tex> — сторона поверхности; ориентированы согласованно с <tex>\partial D</tex><br>
 +
<tex>(P,Q,R)</tex> — гладкое векторное поле на <tex>D</tex>. Тогда:
 +
:<tex dpi=150>\displaystyle\int_{\partial D} P dx + Q dy + R dz = \displaystyle\iint_D (R'_y - Q'_z) \;dy dz + (P'_z - R'_x) \;dz dx + (Q'_x - P'_y) \;dx dy</tex>
 
|proof=
 
|proof=
 
}}
 
}}
=== Формула Гаусса--Остроградского ===
+
 
 +
=== Формула Гаусса Остроградского ===
 
{{Теорема
 
{{Теорема
 
|statement=
 
|statement=
 +
<tex>D \subset \mathbb R^3 \quad \partial D</tex> — ориент. полем внешних нормалей,<br>
 +
<tex>(P, Q, R)</tex> — гл. век. поле в <tex>D</tex>. Тогда
 +
: <tex>\displaystyle\iint\limits_{\partial D} P \,dy\,dz + Q \,dz\,dx + R \,dx\,dy = \iiint\limits_D (P'_x + Q'_y + R'_z)\,dx\,dy\,dz</tex>
 
|proof=
 
|proof=
 
}}
 
}}
 +
 
=== Бескоординатное определение ротора ===
 
=== Бескоординатное определение ротора ===
 
{{Теорема
 
{{Теорема

Текущая версия на 19:14, 12 апреля 2016

Содержание

Определения

Условие L_loc

Определение:
[math]\exists U(y_0)[/math] и [math]\exists g(x)[/math] — суммируемая, что [math]\forall y \in U(y_0) \quad \forall x : |f(x,y)| \le g(x)[/math]
Тогда [math]f[/math] удовлетворяет [math]L_{loc}[/math] в точке [math]y_0[/math]


Образ меры при отображении

Определение:
Пусть [math]\Phi^{-1}(\mathfrak B) \subset \mathfrak A[/math]

[math]\nu \colon \mathfrak B \to \overline{\mathbb{R}}, \quad \nu(\mathfrak B) = \mu(\Phi^{-1}(\mathfrak B))[/math] — мера

[math]\nu[/math] — образ меры [math]\mu[/math] при отображении [math]\Phi[/math]


Взвешенный образ меры

Определение:
[math](X, \mathfrak{A}, \mu), \quad (Y, \mathfrak{B}, ?)[/math]

[math]w \geqslant 0[/math] — измеримая на [math]X[/math] функция
[math]\Phi \colon X \to Y, \quad \Phi^{-1}(\mathfrak B) \subset \mathfrak A[/math]

Тогда [math]\nu(B) = \displaystyle\int\limits_{\Phi^{-1}(\mathfrak B)} w \,d\mu[/math] — взвешенный образ [math]\mu[/math] при отображении [math]\Phi[/math], [math]w[/math] — вес


Плотность одной меры по отношению к другой

Определение:
[math](X, \mathfrak{A}, \mu), \quad (Y, \mathfrak{B}, \nu) [/math]

[math]X = Y, \quad \mathfrak{A} = \mathfrak{B}, \quad \Phi = id[/math]
[math]w \geqslant 0[/math] — вес, измерим на [math]X[/math], [math]f[/math] — изм. на [math]X[/math]
[math]\nu(B) = \int\limits_B w(x) d\mu[/math]

Тогда [math]w[/math] — плотность [math]\nu[/math] относительно [math]\mu[/math]


Заряд

Определение:
[math](X, \mathfrak{A}, \mu), \quad \mu\colon \mathfrak A \to \mathbb{R}[/math] не обязательно [math]\geqslant 0[/math] и обладает свойством счётной аддитивности
Тогда [math]\mu[/math] — заряд


Множество положительности заряда

Определение:
[math]\forall E \in B \ (B \in \mathfrak A) \quad \mu E \geqslant 0[/math] (заряд [math]E[/math] неотрицателен)
[math]B \in \mathfrak A[/math] — множество положительности


Мера, абсолютно непрерывная по отношению к другой мере

Определение:
[math]\mu, \nu \colon \mathfrak A \to \mathbb{R}, \quad \forall a \in \mathfrak A: \mu (a) = 0 \Rightarrow \nu (a) = 0[/math]
Тогда [math]\nu[/math] — абсолютно непрерывная по отношению к мере [math]\mu[/math]


Произведение мер

Определение:
[math](X, \mathfrak{A}, \mu), \quad (Y, \mathfrak{B}, \nu)[/math]

[math]X \times Y[/math] — декартово произведение, [math]\mathfrak{A} \times \mathfrak{B} = \{a \times b \mid a \in \mathfrak{A}, b \in \mathfrak{B}\}[/math]
[math]m \colon A \times B \to R^+, \quad m(a \times b) = \mu(a) \cdot \nu(b)[/math]

[math]m[/math] — произведение мер [math]\mu, \nu[/math] в [math](X \times Y, \mathfrak{A} \times \mathfrak{B}, m)[/math]


Сечение множества

Определение:
Пусть [math]C \subset X \times Y[/math]

[math]C_x = \{y \in Y | (x, y) \in C\}[/math] - сечение [math]C[/math] по [math]X[/math]

[math]C_y = \{x \in X | (x, y) \in C\}[/math] - сечение [math]C[/math] по [math]Y[/math]


Функция распределения

Определение:
[math](X, \mathfrak{A}, \mu)[/math]

[math]h: X \to \mathbb{R}, \quad X(h(x) \lt a)[/math] - конечно

[math]H(a) = \mu X (h(x) \lt a)[/math] - функция распределения [math](: \mathbb{R} \to \mathbb{R})[/math]


Интегральные неравенства Гёльдера и Минковского

Теорема (Гёльдер):
[math](X, \mathfrak{A}, \mu)[/math] — пространство с мерой; [math]f \in L^p, g \in L^q, p \gt 1, \dfrac{1}{p} + \dfrac{1}{q} = 1[/math]. Тогда [math] \displaystyle\int\limits_X |fg| \, d\mu \lt +\infty ,\; \displaystyle\int\limits_X \left|fg\right| \, d\mu \leq \left(\displaystyle\int\limits_X |f|^{p} \, d\mu\right)^{1/p} \left(\displaystyle\int\limits_X |g|^{q} \, d\mu\right)^{1/q}[/math]
Теорема (Минковский):
Пусть [math](X,\mathfrak{A},\mu)[/math] — пространство с мерой, и функции [math]f,g \in L^{p}(X,\mathfrak{A},\mu)[/math]. Тогда [math]f+g \in L^p(X,\mathfrak{A},\mu)[/math], и более того:
[math]\left(\displaystyle\int\limits_X |f(x) + g(x)|^p\, \mu(dx) \right)^{1/p} \leqslant \left( \displaystyle\int\limits_X |f(x)|^p\, \mu(dx)\right)^{1/p} + \left( \displaystyle\int\limits_X |g(x)|^p\, \mu(dx)\right)^{1/p}[/math].

Интеграл комплекснозначной функции

Теорема:
[math]f \colon \mathbb R \to \overline{\mathbb C}[/math]

[math](X, \mathfrak A, \mu)[/math]. Тогда:

  1. [math]f[/math] — изм., если [math]\operatorname{Re}(f), \operatorname{Im}(f)[/math] — изм.
  2. [math]\displaystyle\int_X f\;d\mu = \int_X \operatorname{Re}(f) \;d\mu + i\int_X \operatorname{Im}(f)\;d\mu[/math]
    [math]f[/math] — сумм., [math]\operatorname{Re}(f), \operatorname{Im}(f)[/math] — сумм.

Пространство $L^p(E,\mu)$

Определение:
[math]L^0(E, \mu)[/math] — множество измеримых функций, почти везде конечных на [math]E[/math].


Определение:
[math]L^p(E, \mu) = \Bigl\{f \in L^0(E, \mu) \ \Bigm|\ \displaystyle\int\limits_E |f|^p \;d\mu \lt +\infty \Bigr\}[/math].


Пространство $L^\infty(E,\mu)$

Определение:
[math]L^\infty(E, \mu) = \Bigl\{ f \in L^0(E, \mu) \ \bigl|\ \operatorname*{ess\,sup}\limits_E |f| \lt +\infty \Bigr\}[/math]


Существенный супремум

Определение:
[math] f \colon X \to \overline{\mathbb R}[/math]
[math]\mathrm{ess } \sup f = \inf \{ M \in \overline{\mathbb R} \mid f(x) \leqslant M[/math] при почти всех [math]x\}[/math]


Фундаментальная последовательность, полное пространство

Определение:
Последовательность [math]\{f_n\}_{n \geqslant 1} \subset L^p(X, \mu)[/math] называется фундаментальной в [math]L^p(X, \mu)[/math], если [math]\|f_n - f_k\|_p \to 0[/math] при [math]k, n \to \infty[/math], т.е.
[math]\forall \varepsilon \gt 0 \ \exists N : \|f_n - f_k\| \lt \varepsilon[/math] при [math]k, n \gt N[/math].


Плотное множество

Определение:
[math]X[/math] — метрическое пространство.

[math]A \subset X[/math] — (всюду) плотно в [math]X[/math], если для любого открытого мн-ва [math]G \subset X \quad A \cap G \ne \varnothing[/math].

Или, эквивалентно, любой шар [math]B(x_0, r)[/math] содержит точки из [math]A[/math].


Финитная функция

Определение:
[math]f[/math] — финитная в [math]\mathbb R^m[/math], если она равна нулю вне некоторого шара.


Гильбертово пространство

Определение:
[math]\mathcal H[/math] — полное (любая фундаментальная последовательность сходится в этом пространстве) линейное пространство со скалярным произведением. Под полнотой понимается полнота относительно метрики, порождённой скалярным произведением.


Определение:
[math]\mathcal{H} \[/math] — гильбертово пространство:
  • [math]\forall x, y \in \mathcal H \quad x \perp y \Leftrightarrow \langle x, y \rangle = 0[/math]
  • [math]\mathcal A \in \mathcal H \quad x \perp \mathcal A : \ \forall a \in \mathcal A \ x \perp a[/math]
  • [math]\displaystyle\sum_{k=1}^\infty x_k[/math] — ортогональный ряд, если [math]\forall i, j (i \ne j) \ x_i \perp x_j[/math]


Ортогональная система, ортонормированная система векторов, примеры

Определение:
Система векторов [math]\{e_i\}[/math] называется ортогональной, если [math]\forall i, j \ e_i \perp e_j[/math]


Определение:
Если к тому же [math]\forall i \ |e_i| = 1[/math] — тогда ортонормированная система


Пример:
Стандартный базис евклидового пространства — ортонормированная система


Пример:
[math]\{1, \sin x, \cos x, \sin 2x, \cos 2x, \dotsc\}[/math] — ортогональная система. [math]\left\{\dfrac{1}{\sqrt{2\pi}}, \dfrac{\sin x}{\sqrt \pi}, \dotsc\right\}[/math] — ортонормированная система в [math]L^2[0; 2\pi][/math]


Пример:
[math]1, \left\{\dfrac{e^{ikx}}{\sqrt{2\pi}}\right\}[/math] — ортонормированная система в [math]L^2[0; 2\pi][/math] над [math]\mathbb C[/math]

Сходящийся ряд в гильбертовом пространстве

Определение:
Ряд сходится, если существует элемент из гильбертового пространства, являющийся пределом частичных сумм.


Коэффициенты Фурье, ряд Фурье

Определение:
[math]t \in L^1[-\pi; \pi][/math], тогда [math]a_k, b_k, c_k[/math] — коэффициенты Фурье для [math]t (a_k(f), b_k(f), c_k(f))[/math], а ряд [math]\dfrac{a_0(t)}{2} + \sum a_k(t) \cos kx + b_k(t) \sin kx \ ; \sum c_k(t) e^{ikt}[/math] — ряд Фурье


Базис, полная, замкнутая ОС

Определение:

  1. [math]\{e_k\}[/math] — ОС — базис, если [math]\forall x \in H \quad x = \sum\limits_{k=1}^{+\infty} c_k(x) e_k[/math]
  2. [math]\{e_k\}[/math] — ОС — полная в [math]H[/math], если [math]\left(\forall k\ z \perp e_k\right) \Rightarrow z = 0[/math]
  3. [math]\sum |c_k(x)|^2 \|e_k\|^2 = \|x\|^2[/math] — уравнение Парсеваля (уравнение замкнутости).
    Если [math]\forall x[/math] выполнено уравнение замкнутости, то [math]\{e_k\}[/math] — замкнутая ОС.


Тригонометрический ряд

Определение:
[math]T_n(x) = \dfrac{a_0}{2} + \displaystyle\sum_{k=1}^n a_k \cos kx + b_k \sin kx[/math] — тригонометрический полином степени [math]n[/math].


Определение:
[math]T(x) = \dfrac{a_0}{2} + \displaystyle\sum_{k=1}^{+\infty} a_k \cos kx + b_k \sin kx[/math] — тригонометрический ряд.


Коэффициенты Фурье функции

Определение:
Коэффициенты Фурье функции [math]f[/math][math]a_0(f), a_k(f), b_k(f), c_k(f)[/math] из формулы тригонометрического ряда.

Можно вычислить по формулам:

[math] a_0 = \dfrac{1}{\pi} \cdot \displaystyle\int^\pi_{-\pi} f(x) \,dx \\ a_k = \dfrac{1}{\pi} \cdot \displaystyle\int^\pi_{-\pi} f(x) \cos kx \,dx \\ b_k = \dfrac{1}{\pi} \cdot \displaystyle\int^\pi_{-\pi} f(x) \sin kx \,dx \\ c_k = \dfrac{1}{2\pi} \cdot \displaystyle\int^\pi_{-\pi} f(x) \exp(-ikx) \,dx [/math]


Ядро Дирихле, ядро Фейера

Определение:
[math]D_n(t) = \dfrac{1}{\pi} \left(\dfrac12 + \sum\limits_{k=1}^n \cos kt \right) \quad n = 0, 1, \dotsc[/math] — ядро Дирихле,
[math]\Phi_n(t) = \dfrac{1}{n+1} \sum\limits_{k=0}^n D_k(t)[/math] — ядро Фейера


Свёртка

Определение:
[math]f, k \in L^1[-\pi; \pi][/math]

[math](f*k)(x) = \int\limits_{-\pi}^{\pi} f(t) k(x-t) \;dt = \int\limits_{-\pi}^{\pi} f(x-t) k(t) \;dt[/math]

[math](f*k)(x)[/math] — свёртка.


Аппроксимативная единица

Определение:
[math]D \subset \mathbb R, x_0 \in \overline{\mathbb R}[/math] — пред. точка [math]D[/math].

[math]\forall h \in D[/math] определена функция [math]K_h(x)[/math], удовлетворяющая свойствам:

  • [math]\forall h \in D \ K_h \in L^1[-\pi; \pi] \quad \left(\int\limits_{-\pi}^\pi K_h(t) = 1\right)[/math]
  • L-нормы [math]K_h[/math] огр. в совокупности: [math]\exists M \, \forall h \in D \quad \int\limits_{-\pi}^{\pi} |K_h| \;dt \leqslant M[/math]
  • [math]\forall \delta \gt 0 \int\limits_{E\delta} |K_h| \xrightarrow[h \to x_0]{} 0[/math]
Тогда семейство [math]K_h[/math] называется аппроксимативной единицей.


Усиленная аппроксимативная единица

Определение:
Заменим последнюю аксиому в предыдущем определении на следующую:
[math]K_n \in L^\infty [-\pi; \pi], \quad \operatorname*{ess\,sup}\limits_{E\delta} |K_h| \xrightarrow[h \to x_0]{} 0[/math]
Тогда [math]K_h[/math] — усиленная аппроксимативная единица.


Метод суммирования средними арифметическими

Определение:
[math]\sum a_n = \lim\limits_{n \to \infty} \dfrac{1}{n+1} \cdot \sum\limits_{k=0}^n S_k[/math]


Измеримое множество на простой двумерной поверхности в R^3

Мера Лебега на простой двумерной поверхности в R^3

Определение:
[math]\varphi \colon \mathbb R^2 \to M \subset \mathbb R^3[/math].
Мера в [math]M[/math] — взвешенный образ меры Лебега в [math]\mathbb R^2[/math] с весом [math]|\varphi'_u \times \varphi'_v|[/math]


Поверхностный интеграл первого рода

Определение:
[math]\int f(x(t), y(t), z(t)) \sqrt{x'^2 + y'^2 + z'^2} dt[/math]


Кусочно-гладкая поверхность в ℝ3

Определение:
[math]M \subset \mathbb R^3[/math] называется кусочно-гладкой, если [math]M[/math] представляет собой объединение:
  • конечного числа простых гладких поверхностей
  • конечного числа простых гладких дуг
  • конечного числа точек


Сторона поверхности

Определение:
Сторона поверхности — это непрерывное поле единичных нормалей на поверхности


Задание стороны поверхности с помощью касательных реперов

Определение:
Репер — упорядоченный набор из двух (неколлинеарных) касательных векторов к поверхности


Определение:
Поле реперов [math]v_1, v_2 \colon M \to \mathbb R^3[/math], если [math]\forall x \in M \quad \langle v_1(x), v_2(x) \rangle[/math] — касательный репер


Определение:
Сторона поверхности задаётся с помощью касательных реперов: [math]n_0(x) = \dfrac{v_1(x) \times v_2(x)}{|v_1(x) \times v_2(x)|}[/math]


Интеграл II рода

Определение:
[math] \gamma \colon [a, b] \to \mathbb R^m, \quad V = (A_1, \dotsc, A_m) \\ \displaystyle\int_\gamma A_1 \,dx_1 + \dotsb + A_m \,dx_m = \int_a^b \langle V, \gamma' \rangle \,dt [/math]


Ориентация контура, согласованная со стороной поверхности

Определение:
Ориентация контура называется согласованной со стороной поверхности, если векторное произведение нормали и вектора скорости направлено внутрь контура.


Ротор, дивергенция векторного поля

Определение:
Пусть [math]V = (P, Q, R)[/math] — гладкое векторное поле в некоторой области [math]E \subset \mathbb R^3[/math]. Тогда
[math]\operatorname{rot} V = (R'_y - Q'_z,\; P'_z - R'_x,\; Q'_x - P'_y)[/math]


Соленоидальное векторное поле

Определение:
[math]v = (P, Q, R)[/math] — соленоидальное, если существует векторный потенциал [math]B[/math], т.е. [math]v = \operatorname{rot} B[/math].


Теоремы

Теорема об интегрировании положительных рядов

Теорема:
[math](X, \mathfrak{A}, \mu) \quad U_n - [/math] измеримые функции на [math]X, U_n(x) \geqslant 0 [/math] при почти всех [math]x[/math]. Тогда
[math]\displaystyle\int\limits_X \Bigl(\displaystyle\sum U_n(x)\Bigr) d\mu = \displaystyle\sum \Bigl(\displaystyle\int\limits_X U_n(x) d\mu\Bigr)[/math]
Доказательство:
[math]\triangleright[/math]

Пусть [math]f_n(x) = U_1(x) + U_2(x) + \dotsb + U_n(x)[/math], далее по т. Леви
[math]f = \lim f_n[/math]
[math]0 \leqslant f_n \leqslant f_{n+1} \leqslant \dotsb[/math]

Тогда выражение слева от знака равенства равно [math]\displaystyle\int\limits_X f \,d\mu[/math], а справа — [math] \displaystyle\lim \int\limits_X \sum_{k=1}^n U_k(x) \,d\mu = \lim_{n \to +\infty}\Bigl(\int\limits_X f_n \,d\mu\Bigr) = \int\limits_X f \,d\mu[/math]
[math]\triangleleft[/math]

Абсолютная непрерывность интеграла

Теорема:
[math](X, \mathfrak{A}, \mu), f - [/math] суммируемая функция
[math]\forall \epsilon \gt 0 \quad \exists \delta \gt 0 : \forall E \in \mathfrak{A} \quad \mu E \lt \delta \Rightarrow \int\limits_E |f|d\mu \lt \epsilon[/math]
Доказательство:
[math]\triangleright[/math]

[math]X_n = X (|f| \gt n) \quad X_n \supset X_{n+1} \supset ... \quad \bigcap X_n = e[/math], т.к. [math]f[/math] - суммируема, [math]\mu e = 0[/math]
[math]\nu E = \int\limits_E |f| d\mu[/math] - мера [math]\nu[/math]
[math]\nu X \lt + \infty[/math] (т.к. [math]f[/math] - суммируема и [math]\int\limits_X |f| d\mu \lt +\infty[/math])
Тогда по свойству непрерывности меры сверху: [math]\nu X_n \to 0[/math]
Запишем данное высказывание как [math]\forall \epsilon \gt 0 \quad \exists n_\epsilon : \nu(X_{n_\epsilon}) \lt \dfrac{\epsilon}{2}[/math], т.е. [math]\int\limits_{X_{n_\epsilon}} |f| \lt \dfrac{\epsilon}{2}[/math]
Теперь пусть [math]\delta := \dfrac{\epsilon}{2 \cdot n_\epsilon}[/math]

[math]\int\limits_E |f| d\mu = \int\limits_{E \cap X_{n_\epsilon}} |f| d\mu + \int\limits_{E \cap X^C_{n_\epsilon}} |f| d\mu \leqslant \int\limits_{X_{n_\epsilon}} |f| d\mu + \int\limits_{E \cap X^C_{n_\epsilon}} n_\epsilon d\mu \leqslant \dfrac{\epsilon}{2} + n_\epsilon \cdot \mu E \lt \epsilon[/math]
[math]\triangleleft[/math]

Теорема Лебега о мажорированной сходимости для случая сходимости по мере

Теорема:
[math](X, \mathfrak{A}, \mu), f, f_n: X \rightarrow \mathbb{R}, f_n \rightarrow f[/math] по мере [math]\mu[/math]

[math]\exists g[/math] - суммируемая и [math]\forall n |f_n| \leqslant g[/math] для почти всех [math]x[/math]

Тогда [math]f_n, f[/math] - суммируемые и [math]\int |f-f_n| d\mu \to 0[/math]
Доказательство:
[math]\triangleright[/math]

[math]f_n[/math] - суммируема, т.к. [math]\int |f_n| \leqslant \int g \lt + \infty[/math]
[math]f[/math] - суммируема, т.к. [math]\exists f_{n_k} \to f[/math] почти везде, [math] |f_{n_k}| \leqslant g \Rightarrow |f| \leqslant g[/math]
[math]\int\limits_X |f_n - f| d\mu \to 0 ?[/math]
Рассмотрим два случая:
1) [math]\mu X \lt +\infty[/math]
Берём [math]\epsilon \gt 0 \quad X_n := X (|f_n - f| \gt \epsilon) \quad \mu X_n \to 0[/math]
[math]\int\limits_X |f_n - f| d\mu \leqslant \int\limits_{X_n} |f_n - f| d\mu + \int\limits_{X^C_n} |f_n - f| d\mu[/math]
Для [math]X_n[/math] выполнено [math]|f_n - f| \leqslant |f_n| + |f| \leqslant 2 \cdot g[/math]
А для [math]X^C_n[/math] выполнено [math] |f_n - f| \lt \epsilon[/math]
Тогда [math]\int\limits_{X_n} |f_n - f| d\mu + \int\limits_{X^C_n} |f_n - f| d\mu \leqslant \int\limits_{X_n} 2 \cdot g d\mu + \int\limits_{X^C_n} \epsilon d\mu \leqslant 2 \cdot \int\limits_{X_n} g + \epsilon \cdot \mu X \leqslant \epsilon \cdot (2 + \mu X)[/math]
Получили [math]\forall \epsilon \gt 0 \quad \exists N: \forall n \gt N \quad \int\limits_X |f_n - f| d\mu \lt \epsilon \cdot (2 + \mu X)[/math]
Осталось найти номер [math]N[/math]. Нужно взять такой, чтобы [math]\mu X_n \lt \delta[/math].
2) [math]\mu X = +\infty[/math]

TBD
[math]\triangleleft[/math]

Теорема Лебега о мажорированной сходимости для случая сходимости почти везде

Теорема:
[math](X, \mathfrak{A}, \mu), f, f_n \colon X \rightarrow \overline{\mathbb{R}}, f_n \rightarrow f [/math] почти везде

[math]\exists g[/math] - суммируемая и [math]\forall n |f_n| \leqslant g[/math] для почти всех [math]x[/math]

Тогда [math]f_n, f[/math] суммируемые и [math]\displaystyle\int |f-f_n|d\mu \to 0, \int_X f_n \to \int_X f[/math]
Доказательство:
[math]\triangleright[/math]

Легко видеть, что [math]f, f_n[/math] — суммируемые.
[math] h_n := \sup(|f_n - f|, |f_{n+1} - f|, \dotsc) \\ h_n \geqslant h_{n+1} \geqslant \dotsb; \qquad |f_n - f| \leqslant 2g \Rightarrow h_n \leqslant 2g [/math]

Кстати, [math]\lim h_n = \varlimsup |f_n - f| = 0[/math] при п.в. [math]x[/math].

Рассмотрим ф-ии [math]2g - h_n \geqslant 0[/math] — возр.

[math]\lim \displaystyle\int_X (2g - h_n) = \int_X \lim(2g - h_n) = 2 \int_X g[/math]

С другой стороны,

[math]\lim \displaystyle\int_X (2g - h_n) = \lim\biggl(2 \int_X g - \int_X h_n\biggr) \Rightarrow \int_X h_n \to 0 \Rightarrow \int_X |f_n - f| \leqslant \int_X h_n[/math]
[math]\triangleleft[/math]

Теорема Фату

Теорема:
[math](X, \mathfrak{A}, \mu), f_n \to f[/math] почти везде на [math]X[/math], и [math]\exists C \gt 0: \forall n \displaystyle\int_X {f_n \;d\mu} \leqslant C[/math]
Тогда [math]\displaystyle\int f \;d\mu \leqslant C[/math]
Доказательство:
[math]\triangleright[/math]
[math] g_n = \inf(f_n, f_{n+1}, \dotsc)\\ g_n(x) \leqslant g_{n+1}(x) \leqslant \dotsb \quad \lim g_n = \varliminf f_n = f\\ \displaystyle\int g_n \leqslant \int f_n \leqslant C\\ \int f \;d\mu = \lim_{n \to +\infty} \int g_n \leqslant C [/math]
[math]\triangleleft[/math]

Теорема Лебега о непрерывности интеграла по параметру

Теорема:
[math]f: X \times Y \rightarrow \mathbb{R}, \forall y \int\limits_X f(x, y) d\mu(x)[/math] - имеет смысл и выполнены 2 условия:
  1. [math]f[/math] удовлетворяет условию [math]L_{loc}(y_0)[/math]
  2. [math] y \rightarrow f(x, y)[/math] - непрерывна при всех [math]x[/math]
    [math]f(x, y) \rightarrow f(x, y_0)[/math] при [math]y \to y_0[/math] при всех [math]x[/math]
    Тогда [math]I(y) = \int\limits_X f(x, y) d\mu(x)[/math] непрерывна в [math]y_0[/math]
Доказательство:
[math]\triangleright[/math]

Рассмотрим [math]f_n(x) = f(x, y_n)[/math], где [math]y_n \rightarrow y_0, y_n \in (Y \cap U) \setminus \{a\}[/math].

Применим теорему Лебега для [math]f_n[/math].
[math]\triangleleft[/math]

Правило Лейбница дифференцирования интеграла по параметру

Теорема:
[math]f: X \times Y \rightarrow \mathbb{R}, Y \in \mathbb{R}[/math] - промежуток
  1. [math]\forall y \quad x \rightarrow f(x, y)[/math] - суммируема, [math]I(y) = \int\limits_X f(x, y) d\mu(x)[/math]
  2. [math]\forall y[/math] при всех [math]x \quad \exists^* f'_y(x, y)[/math]
  3. [math]y_0 \in Y \quad f'_y(x, y)[/math] удовлетворяет условию [math]L_{loc}(y_0)[/math]
    Тогда [math]I'(y_0) = \int\limits_X f'_y(x, y)d\mu(x)[/math]
Доказательство:
[math]\triangleright[/math]

Пусть [math]x \in X, y_0 + h \in Y, h \not = 0[/math]
[math]F(x, h) = \frac{f(x, y_0 + h) - f(x, y_0)}{h}[/math]
Т.к. [math]\frac{I(y_0 + h) - I(y_0)}{h} = \int\limits_X \frac{f(x, y_0 + h) - f(x, y_0)}{h} d\mu(x) = \int\limits_X F(x, h) d\mu(x)[/math], то при [math]h \rightarrow 0[/math] сразу будет следовать теорема. Для доказательства законности этого перехода докажем, что [math]F[/math] удовлетворяет [math]L_{loc}[/math] в [math]h = 0[/math]:

[math]f'_y[/math] удовлетворяет условию [math]L_{loc}[/math], поэтому найдутся такие [math]\delta[/math] и [math]g[/math], что [math]|f'_y(x, y)| \leq g(x)[/math] при почти всех [math]x[/math] и при [math]y \in Y, 0 \lt |y - y_0| \lt \delta[/math].

Теорема Лагранжа о среднем применённая к [math]y \rightarrow f(x, y)[/math] на [math](y_0, y_0 + h)[/math] даст [math]F(x, h) = f'_y(x, y_0 + \theta h)[/math]. Поэтому [math]F(x, h) \leq g(x)[/math].
[math]\triangleleft[/math]

Вычисление интеграла Дирихле

Теорема:
[math]\displaystyle\int\limits_0^{+\infty} \dfrac{\sin \alpha x}{x} = \dfrac{\pi}{2} \cdot \operatorname{sgn}(\alpha)[/math]
Доказательство:
[math]\triangleright[/math]
Можно, например, вот так.
[math]\triangleleft[/math]

Теорема о вычислении интеграла по взвешенному образу меры

Теорема:
[math] (X, \mathfrak{A}, \mu), (Y, B, ???)[/math]

[math] w \geqslant 0 [/math] - измеримая на [math]X[/math] функция
[math] \phi: X \rightarrow Y \quad \phi^{-1}(B) \in A[/math]
[math]v(B) = \displaystyle\int\limits_{\phi^{-1}(B)} w(x) d\mu[/math] - взвешенный образ [math]\mu[/math] при отображении [math]\phi, w [/math] - вес

Тогда: [math]\forall Y_0 \in Y \displaystyle\int\limits_{Y_0} f(y) dv = \int\limits_{\phi^{-1}(Y_0)} f(\phi(x)) \cdot w(x) d\mu(x)[/math]
Доказательство:
[math]\triangleright[/math]

Это очевидно верно, если [math]f -[/math] характеристическая функция. По линейности интеграла это также верно и для простой неотрицательной [math]f[/math].

Для произвольной неотрицательной [math]f[/math] рассмотрим последовательность простых неотрицательных функций [math]f_n[/math] и по теореме Леви (предельный переход) теорем доказана для неотрицательных [math]f[/math].

Для отрицательных там надо что-то ещё сделать))))
[math]\triangleleft[/math]

Критерий плотности

Теорема:
[math](X, \mathfrak(A), \mu) \quad v, w[/math] - измеримые, [math]w \geqslant 0[/math]
[math]w [/math] - плотность [math]v[/math] относительно [math]\mu \Leftrightarrow \forall T \in A \quad \mu(T) \times \inf(w) \leqslant v(T) \leqslant \mu(T) \times \sup(w)[/math]
Доказательство:
[math]\triangleright[/math]

[math]\Rightarrow)[/math] Очевидно
[math]\Leftarrow)[/math] Пусть [math]w \gt 0[/math] (без потери общности)
[math]A = \bigcup\limits_{k \in \mathbb{Z}} A_k (q^k \leqslant w \leqslant q^{k-1}) \quad q \in (0, 1)[/math]
[math]q^k \cdot \mu A_k \leqslant \nu (A_k) \leqslant q^{k-1} \cdot \mu A_k[/math]
[math]q^k \cdot \mu A_k \leqslant \int\limits_{A_k} w d\mu \leqslant q^{k-1} \cdot \mu A_k[/math]
[math]q \cdot \int\limits_{A_k} w d\mu \leqslant q^k \cdot \mu A_k \leqslant \nu(A_k) \leqslant \dfrac{1}{q} \cdot q^k \cdot \mu(A_k) \leqslant \dfrac{1}{q} \cdot \int\limits_{A_k} w d\mu[/math]
[math]q \cdot \int\limits_{A_k} w d\mu \leqslant \nu(A_k) \leqslant \dfrac{1}{q} \cdot \int\limits_{A_k} w d\mu[/math]
[math]q \to 1-0[/math]

[math]\nu(A) = \int\limits_{A} w d\mu[/math]
[math]\triangleleft[/math]

Лемма о множествах вполне положительности заряда

Теорема:
[math](X, \mathfrak{A}, \mu), \quad A \in \mathfrak A, \quad \mu A \geqslant 0[/math]
Тогда [math]\exists B \subset A[/math] — множество положительности: [math]\mu(B) \geqslant \mu(A)[/math]
Доказательство:
[math]\triangleright[/math]
Определение:
[math]C[/math] — мн-во [math]\varepsilon[/math]-положительности,если [math]\forall B \subset C \quad \mu B \geqslant -\varepsilon[/math]
Утверждение:
[math]\forall \varepsilon \gt 0 \ A[/math] содержит мн-во [math]\varepsilon[/math]-положительности.
[math]\triangleright[/math]
  1. [math]A[/math] — мн-во [math]\varepsilon[/math]-положительности — очевидно
  2. [math]A[/math] не явл. мн-вом [math]\varepsilon[/math]-положительности: [math]\exists B_1 \subset A : \mu B_1 \lt -\varepsilon[/math]
    [math]C_1 := A \setminus B_1 \Rightarrow \mu C_1 \gt \mu A[/math]
    1. [math]C_1[/math] — мн-во [math]\varepsilon[/math]-положительности — ОК
    2. Иначе [math]\exists B_2 \subset C_1 : \mu B_2 \lt -\varepsilon \quad C_2 := C_1 \setminus B_2 \quad \mu C_2 \gt \mu C_1[/math]
  3. Продолжаем в том же духе — и рано или поздно приходим к успеху, т.к. иначе [math]\mu \left( \bigcup B_i \right) = -\infty[/math]
[math]\triangleleft[/math]

[math]C_1 \subset A[/math] — мн-во 1-положительности:   [math]\mu C_1 \geqslant \mu A[/math]
[math]C_2 \subset C_1[/math] — мн-во [math]1/2[/math]-положительности:   [math]\mu C_2 \geqslant \mu C_1[/math]
[math]\vdots[/math]
[math]C_n \subset C_{n-1}[/math] — мн-во [math]1/n[/math]-положительности:   [math]\mu C_n \geqslant \mu C_{n-1}[/math]
Пусть [math]B = \bigcap C_i[/math]

[math]\mu B = \lim\limits_{i \to +\infty} \mu C_i \geqslant \mu A[/math]
[math]\triangleleft[/math]

Теорема Радона — Никодима

Теорема (Радон, Никодим):
[math](X, \mathfrak{A}, \mu)[/math] — пространство с мерой, [math]\nu \colon \mathfrak{A} \to \mathbb R, \quad \mu, \nu[/math] — конечные меры, причём [math]\nu[/math] абсолютно непрерывна относительно [math]\mu[/math].

Тогда [math]\exists ! f[/math] — сумм. отн. [math]\mu[/math]

[math]f[/math] — плотность [math]\nu[/math] относительно [math]\mu[/math].
Доказательство:
[math]\triangleright[/math]

Единственность

Лемма:
Если [math]f, g[/math] — сумм. отн. [math]\mu[/math] и [math]\displaystyle \forall A \in \mathfrak{A} \int_A f \, d\mu = \int_A g \, d\mu[/math], то [math]f = g[/math] п.в.
Доказательство:
[math]\triangleright[/math]

[math]h := f - g[/math].

[math] \forall A \in \mathfrak A \quad \displaystyle\int_A h \,d\mu = 0 \\ \\ X = X(h \geqslant 0) \cup X(h \lt 0) \\ \\ \int\limits_{h \geqslant 0} h \,d\mu = 0, \quad \int\limits_{h \lt 0} h \,d\mu = 0 [/math]

Легко видеть, что [math]\displaystyle\int_X |h| \,d\mu = 0 \ \Rightarrow h = 0[/math] п.в.
[math]\triangleleft[/math]

Существование

TBD
[math]\triangleleft[/math]

Лемма об оценке мер образов кубов из окрестности точки дифференцируемости

Теорема:
[math]\phi: O \in \mathbb{R}^m \to \mathbb{R}^m, \quad a \in \mathbb{R}^m, f[/math] - диффиренцируема в [math]a[/math]

Пусть [math]c \gt |\det \varphi'(a)| \gt 0, \quad \mu[/math] - мера Лебега на [math]\mathbb{R}^m[/math]
Тогда [math]\exists U(a) \quad \forall[/math] куба [math]Q \subset U(A), a \in Q[/math]

[math]\mu(\phi(Q))\lt c \cdot \mu(Q)[/math]

Теорема о преобразовании меры при диффеоморфизме

Теорема:
[math]\phi \colon O \in \mathbb{R}^m \to \mathbb{R}^m[/math] - диффеоморфизм
Тогда [math]\forall x \in \mathbb{R}^m \mu(\phi(a)) = \int\limits_a |\det \phi'(x)| \cdot d\mu(x)[/math]

Теорема о гладкой замене переменной в интеграле Лебега

Теорема:
[math]\varphi\colon O \subset \mathbb{R}^m \to \mathbb{R}^m [/math] — диффеоморфизм

Пусть [math]O_1 := \varphi(O), \quad f \geqslant 0 [/math] — измерима на [math]O_1[/math]

Тогда [math]\int\limits_{O_1} f(y) d\mu = \int\limits_{O} (f * \varphi)(x) \cdot |\det \varphi'(x)| d\mu(x)[/math]

Теорема о произведении мер

Теорема:
[math]\mathbb{R}^n \Rightarrow \lambda_a \cdot \lambda_b = \lambda_{a+b}[/math]

Принцип Кавальери

Теорема:
[math](X, \mathfrak{A}, \mu), \quad (Y, \mathfrak{B}, \nu) \quad \mu, \nu[/math] - сигма конечные, полные; [math]m = \mu \times \nu[/math]

[math]C[/math] измеримо в [math]\mathfrak{A} \times \mathfrak{B}[/math]
Тогда:

  1. [math]C_x - \mu[/math] — измерима при всех [math]x[/math]
  2. [math]x \mapsto \nu(x)[/math] измерима при всех [math]x[/math]
  3. [math]mc = \int\limits_X \nu(C_x)d\mu(x)[/math]
Аналогично для [math]C_y[/math]

Теорема Тонелли

Теорема:
[math](X, \mathfrak{A}, \mu), \quad (Y, \mathfrak{B}, v) \quad \mu v[/math] - сигма конечные, полные; [math]m = \mu * v[/math]

[math]f: X \times Y \to \mathbb{R}, f \geqslant 0[/math] измеримая, [math]f_x := y \to f(x, y)[/math]
Тогда:

  1. [math]f_x - v[/math]-измерима при почти всех [math]x[/math]
  2. [math]f_y - \mu[/math]-измерима при почти всех [math]y[/math]
  3. [math]x \to \phi(x) := \int f_x dv[/math] - [math] \mu[/math]-измеримая функция
  4. [math]\int\limits_{X \times Y} f dm = \int \limits_X \phi(x) d\mu(x) = \int (\int f(x, y) dv(y)) d\mu(x) = \int (\int f(x, y) d\mu(x)) dv(y)[/math]

Формула для Бета-функции

Теорема:
[math]B(x, y) = \dfrac{\Gamma(x) \cdot \Gamma(y)}{\Gamma(x + y)}[/math]
Доказательство:
[math]\triangleright[/math]

Вычислим интеграл [math]I(u, v) = \displaystyle\iint\limits_{x, y \gt 0} e^{-(x^2 + y^2)} x^{2u - 1} y^{2v-1} \;dx dy[/math]

С одной стороны, [math]I(u, v) = I(u) \cdot I(v)[/math], где

[math]I(u) = \displaystyle\int\limits_0^{+\infty} e^{-x^2} x^{2u-1} \;dx = \dfrac12 \int\limits_0^{+\infty} e^{-t} t^{u-1} \;dt = \dfrac12 \Gamma(u)[/math]

С другой стороны, переходя к полярным координатам, получим:

[math]I(u, v) = \displaystyle\int\limits_0^{+\infty} e^{-r^2} r^{2u + 2v - 1} \;dr \int\limits_0^{\pi/2} \cos^{2u-1} \varphi \sin^{2v-1} \varphi \;d\varphi = {}\\ {} = \dfrac12 \Gamma(u + v) \int\limits_0^{\pi/2} \cos^{2u-1} \varphi \sin^{2v-1} \varphi \;d\varphi[/math]

Сделаем замену [math]\cos^2 \varphi = t[/math]:

[math]\displaystyle\int\limits_0^{\pi/2} \cos^{2u-1} \varphi \sin^{2v-1} \varphi \;d\varphi = \frac{1}{2} B(u, v)[/math]
Составляя два выражения для [math]I(u, v)[/math], получим [math]B(x, y) = \dfrac{\Gamma(x) \cdot \Gamma(y)}{\Gamma(x + y)}[/math]
[math]\triangleleft[/math]

Теорема Фубини

Теорема:
[math](X, \mathfrak{A}, \mu), \quad (Y, \mathfrak{B}, \nu) \quad \mu, \nu[/math] — сигма-конечные, полные; [math]m = \mu \times \nu[/math]

[math]f \colon X \times Y \to \overline{\mathbb{R}}[/math][math]m[/math]-сумм. Тогда:

  1. [math]C_x[/math] — суммируема при всех [math]x[/math]
  2. [math] x \mapsto q(x) = \int f_x \,d\nu[/math] сумм при всех [math]x[/math]
  3. [math]\int f \,d\nu = \int q \,d\mu[/math]
Аналогично для [math]C_y[/math]
Доказательство:
[math]\triangleright[/math]

[math]f = f_+ - f_- \quad \int\limits_{X \times Y} f_\pm \,dm[/math] — кон.
[math]\displaystyle\int_{X \times Y} f \,dm = \int_{X \times Y} f_+ \,dm - \int_{X \times Y} f_- \,dm[/math]
[math]\displaystyle\int(f_x)_+ , \int(f_x)_-[/math] — кон. при п.в. [math]x[/math]
Т.к. [math]f_+ \geqslant 0 \Rightarrow \displaystyle\int_X \left( \int_Y (f_x)_+ \,d\nu \right) d\mu[/math] — кон. [math] \Rightarrow \displaystyle\int_Y (f_x)_+\,d\nu[/math] — кон. при п.в. [math]x[/math]

[math] \varphi(x)_+ = \displaystyle\int_Y (f_x)_+ \,d\nu \\ \varphi(x) = \displaystyle\int_Y (f_x)_+ \,d\nu - \displaystyle\int_Y (f_x)_- \,d\nu \\ \int_X |\varphi(x)| \,d\mu = {} \\ {} = \int_X \left| \int_Y (f_x)_+ - \int_Y (f_x)_- \right| \,d\mu \leqslant \int_X \left( \left| \int_Y (f_x)_+ \right| - \left|\int_Y (f_x)_-\right| \right) \,d\mu \\ \int\limits_{X \times Y} f \,dm = \left(\int\limits_{X \times Y} f_+ \right) - \left(\int\limits_{X \times Y} f_- \right) = \int\limits_X \int\limits_Y f_+ - \int\limits_X \int\limits_Y f_- = {} \\ {} = \int\limits_X \left(\int\limits_Y f_+ - \int\limits_Y f_- \right) = \int\limits_X \int\limits_Y f [/math]
[math]\triangleleft[/math]

Объем шара в R^m

Теорема:
[math]V(B(0, r)) = \alpha \cdot r^n[/math]
[math]\alpha = \dfrac{(\sqrt{\pi})^n}{\Gamma\left(\dfrac{n}{2} + 1\right)}[/math]

Теорема о вычислении интеграла по мере Бореля — Стилтьеса (с леммой)

Лемма:
[math](X, \mathfrak{A}, \mu), \quad h[/math] — измерима, почти везде конечна

[math]H[/math] — функция распределения: [math]H(t) = \mu X (h \lt t)[/math]
[math]\nu = h(\mu)[/math], т.е. [math]\nu(A) = \mu(h^{-1}(A))[/math]
[math]\mu_H[/math] — мера Бореля-Стилтьеса от [math]H[/math]

Тогда [math]\mu_H \equiv \nu[/math] на [math]B[/math] (Борелевской сигма-алгебре)
Доказательство:
[math]\triangleright[/math]

[math][a, b) \quad \mu_H [a;\, b) = H(b-0) - H(a-0) = (*)[/math]
[math]H\left(b - \dfrac1n\right) = \mu X\left(h \lt b - \dfrac1n\right)[/math]
[math]H(b-0) = \lim\limits_{n \to +\infty} \mu X \left(h \lt b - \dfrac1n\right) = \mu X(h\lt b)[/math]   [math]\left(\displaystyle \bigcup X \left(h \lt b - \dfrac1n\right) = X(h\lt b)\right)[/math]

[math](*) = H(b) - H(a) = \mu X(h \lt b) -\mu X(h \lt a) = \mu X(a \leqslant h \lt b) = \mu h^{-1} [a, b)[/math] [math]{ } = \nu [a, b)[/math]
[math]\triangleleft[/math]


Теорема:
[math]f: \mathbb{R} \to \mathbb{R}, \quad f \geqslant 0[/math] измерима относительно [math]B[/math]

Остальное из прошлой леммы

Тогда: [math]\int\limits_X f(h(x)) d\mu(x) = \int\limits_R f(t) d\mu_h(t)[/math]
Доказательство:
[math]\triangleright[/math]
Ну тут тип просто замена в интеграле)))
[math]\triangleleft[/math]

Теорема о вложении пространств L^p

Теорема:
[math](X, \mathfrak{A}, \mu)[/math]

[math]\mu(X) \lt +\infty[/math]

  1. [math]1 \leqslant s \lt r \lt +\infty[/math], тогда [math]L^r \subset L^s[/math]
  2. [math]\| f \|_s \leqslant (\mu(X))^{\frac{1}{s} - \frac{1}{r}} \cdot \| f \|_r[/math]
Доказательство:
[math]\triangleright[/math]
  1. Напрямую следует из 2
  2. Пусть
    [math] \dfrac{r}{s} = p \gt 1[/math]
    [math] q = \dfrac{r}{r - s}[/math]

    Тогда: [math]\| f \|^s_s = \int\limits_X |f|^s = \int\limits_X |f|^s \cdot 1 \leqslant \left(\int\limits_X |f|^{s \cdot \frac{r}{s}}\right)^\frac{s}{r} \cdot \left(\int\limits_X 1^{\frac{r}{r-s}}\right)^\frac{r-s}{r} = \| f \|_r^s \cdot (\mu(X))^{1-\frac{s}{r}}[/math] (по Гёльдеру)

[math]\triangleleft[/math]

Полнота L^p

Теорема:
[math](X, \mathfrak{A}, \mu), L^p(X)[/math] — полное [math](1 \leqslant p \lt +\infty)[/math]
Доказательство:
[math]\triangleright[/math]

[math]f_n[/math] — фундамтельная в [math]L^p[/math]
Строим кандидата на роль предела:
[math]\varepsilon := \dfrac{1}{2} \quad \exists N_1 \quad \forall m, n \geqslant N_1 \quad \|f_m - f_n\|_p \lt \dfrac{1}{2}\\ \\ \varepsilon := \dfrac{1}{4} \quad \exists N_2 \gt N_1 \quad \forall m, n \geqslant N_2 \quad \|f_m - f_n\|_p \lt \dfrac{1}{4}\\ \\ \varepsilon := \dfrac{1}{8} \quad \dots[/math]

Очевидно, что [math]\sum\limits_{k=1}^{+\infty} \|f_{N_{k+1}} - f_{N_k}\| \lt 1[/math]

Рассмотрим [math]S(x) = \sum\limits_{k=1}^{+\infty} |f_{N_{k+1}}(x) - f_{N_k}(x)| \in [0; +\infty][/math]

[math]\|S_N\|_p = \|\sum ... \|_p \leqslant \sum\limits_{k=1}^{N} \|f_{N_{k+1}} - f_{N_k}\|_p \lt 1[/math]

Т.е. [math]\displaystyle\int\limits_X |S_N(x)|^p d\mu(x) \lt 1[/math]
При всех [math]x \quad S_N(x) \to S(x)[/math]

По теореме Фату [math]\displaystyle\int\limits_X |S(x)|^p \lt 1[/math], т.е. [math]|S(x)|^p[/math] - суммируема
Значит [math]|S(x)|[/math] почти везде конечна. [math] \Rightarrow [/math] Ряд [math] \sum f_{N_{k+1}}(x) - f_{N_k}(x)[/math] абсолютно сходится при почти всех [math]x[/math].

[math]f(x) = f_{N_1}(x) + \sum f_{N_{k+1}}(x) - f_{N_k}(x)[/math]
При всех [math]x \quad f(x) = \lim\limits_{k \to +\infty} f_{N_1} + \sum\limits_{i=1}^{k}(...) = \lim\limits_{k \to +\infty} f_{N_{k+1}}(x)[/math]
[math]\|f\|_p \leqslant \|f_{N_1}\|_p + \sum \|f_{N_{k+1}} - f_{N_k}\|_p[/math] — конечна
[math]\|f(x)-f_n(x)\|_p \to 0 ?[/math]

[math]\forall \varepsilon \gt 0 \quad \exists N \quad \forall m, n \gt N \quad \|f_n-f_m\|_p^p \lt \varepsilon^p[/math]
Возьмём [math]m:=N_k \gt N[/math]
[math]\|f_n-f_{N_k}\|_p^p \lt \epsilon^p[/math]
[math]\displaystyle\int\limits_X |f_n(x) - f_{N_k}(x)|^p d\mu(x) \lt \varepsilon^p[/math]

По теореме Фату:

[math]\displaystyle\int\limits_X |f_n(x) - f(x)|^p d\mu \lt \varepsilon^p \Rightarrow f_n \rightrightarrows f[/math]
[math]\triangleleft[/math]

Плотность в L^p множества ступенчатых функций

Теорема:
[math](X, \mathfrak{A}, \mu), f - [/math] ступенчатая  [math]{} = \sum\limits_{k=1}^{n} C_k \times \chi_{E_k}[/math]

[math]X = \bigsqcup X_k[/math]

[math]\mu X (f \neq 0) -[/math] конечно

в [math]L^p(X, \mu) (1 \leqslant p \leqslant +\infty)[/math] множество ступенчатых функций плотно
Доказательство:
[math]\triangleright[/math]
  1. [math]p = \infty \quad f \in L^\infty \quad \|f\|_\infty = \operatorname{ess\,sup} |f| \lt +\infty[/math]
    Поправив [math]f[/math] на множестве нулевой меры, получим [math]\forall x \in X \ |f(x)| \leqslant \|f\|_\infty[/math]
    [math]f[/math] — изм. огр., [math]\exists h_n : \sup |f - h_n| \to 0 \Rightarrow \|f - h_n\|_\infty = \operatorname{ess\,sup} |f - h_n| \leqslant \sup |f - h_n|[/math]
  2. [math]p \lt +\infty \quad f \in L^p \quad B(f, \varepsilon)[/math] — есть ли здесь ступ. ф-ия?
    [math]f \geqslant 0 \quad \exists[/math] ступ. [math]h_n : h_n \leqslant h_{n+1} \leqslant \dots \quad h_n \to f, h_n \leqslant f[/math]
    [math]\displaystyle\int\limits_X |f - h_n| \to 0[/math]
    [math]\|f - h_n\|^p_p = \displaystyle\int\limits_X |f - h_n|^p d\mu(x) \xrightarrow[n \to +\infty]{} 0[/math] (по т. Лебега).
[math]\triangleleft[/math]

Лемма Урысона

Теорема:
[math]F_0, F_1 - [/math] два непересекающихся замкнутых множества из [math]\mathbb{R}^m[/math]
Тогда [math]\exists f: \mathbb{R}^m \to \mathbb{R}[/math] (непрырывная)[math]: f|_{F_0}=0, f|_{F_1}=1[/math]
Доказательство:
[math]\triangleright[/math]

[math]\forall[/math] замкн. [math]F[/math] и [math]\forall[/math] откр. [math]G \supset F[/math] [math]\exists[/math] откр. [math]H : F \subset H \subset \overline H \subset G[/math].
[math]\exists U(F_0), U(F_1)[/math] — откр.: [math]U(F_0) \cap U(F_1) = \varnothing[/math]
[math]F_0 \subset G_0 \subset \overline{G_0} \subset F_1^c = G_1[/math]
[math]\overline{G_0} \subset G_1 \quad \exists G_{1/2} \quad \overline{G_0} \subset G_{1/2} \subset \overline{G_{1/2}} \subset G_1[/math]
Аналогично можно ввести [math]G_{1/4}, G_{3/4}[/math] и так далее [math]G_{\alpha}[/math] для любого двоично-рационального [math]\alpha \in [0; 1][/math].

[math]f(x) := \sup \{x \in G_\alpha \mid \alpha[/math] — дв. рац. [math] \}[/math] — непр.

[math](a, b) \subset [0, 1], a[/math] — дв. рац.   [math]{}\quad f^{-1}(a, b) = \!\!\!\!\!\!\displaystyle\bigcup_{\substack{\alpha \in (a, b) \\ \alpha \text{ is dyadic rat.}}}\!\!\!\!\!\! G_\alpha \setminus \overline{G_a}[/math]
[math]\triangleleft[/math]

Плотность в L^p непрерывных финитных функций

Теорема:
[math]\forall p: 1 \leqslant p \lt +\infty \quad C_0[/math] всюду плотно в [math]L^p(R^m)[/math]

Теорема о непрерывности сдвига

Теорема:
[math]f_n(x) = f(x + h)[/math]
  1. [math]f[/math] - равномерно непрерывна на [math]\mathbb{R}^m \Rightarrow \displaystyle\lim_{h \to 0} \| f_n - f \|_\infty = 0[/math]
  2. [math]1 \leqslant p \lt +\infty \quad f \in L^p (\mathbb{R}^m) \Rightarrow \displaystyle\lim_{h \to 0} \| f_n - f \|_p = 0[/math]
  3. [math]f \in \widetilde{C}[0, T] \Rightarrow \displaystyle\lim_{h \to 0} \| f_n - f \|_\infty = 0[/math]
  4. [math]1 \leqslant p \lt +\infty \quad f \in L^p[0, T] \Rightarrow \lim\limits_{h \to 0} \| f_n - f \|_p = 0[/math]

Теорема о свойствах сходимости в гильбертовом пространстве

Теорема:
Пусть есть ГП
  1. [math]x_n \to x, y_n \to y \quad[/math] Тогда [math]\langle x_n, y_n\rangle \to \langle x, y \rangle[/math]
  2. [math]\displaystyle\sum_{n=1}^{+\infty} x_n - [/math] ряд, сходящийся в ГП. Тогда [math]\forall y \ \bigl\langle y, \sum_{n=1}^{+\infty} x_n \bigr\rangle = \displaystyle\sum_{n=1}^{+\infty} \langle y, x_n \rangle[/math]
  3. [math]\displaystyle\sum_{n=1}^{+\infty} x_n - [/math] ортогональный ряд. Тогда [math]\displaystyle\sum_{n=1}^{+\infty} x_n - [/math] сходится [math]\Leftrightarrow \displaystyle\sum_{n=1}^{+\infty} \| x_n \| - [/math] сходится.

Теорема о коэффициентах разложения по ортогональной системе

Теорема:
[math]\mathcal{H} -[/math] ГП

[math]\{e_k\} - [/math] Ортогональная система. [math]x = \displaystyle\sum_{k=1}^{+\infty} C_k \cdot e_k[/math]

Тогда:

  1. [math]\{e_k\} - [/math] ЛНЗ
  2. [math]\dfrac{\langle x, e_k \rangle}{\|e_k\|^2} = C_k[/math]
  3. [math]C_k \cdot e_k - [/math] это проекция [math]X[/math] на 1-номерное подпространство, порождённое [math]e_k[/math].
[math] x = C_k \cdot e_k + z \Rightarrow z \perp e_k [/math]

Теорема о свойствах частичных сумм ряда Фурье. Неравенство Бесселя

Теорема:
[math]\{e_k\} - [/math] Ортогональная система в [math]\mathcal{H}, x \in \mathcal{H}[/math]

[math]S_n = \displaystyle\sum_{k=1}^{n} C_k (x) \cdot e_k - [/math] частичные суммы ряда Фурье

[math]\alpha_n := \operatorname{Lin}(e_1, \dotsc, e_n)[/math]

Тогда:

  1. [math]S_n - [/math] проекция [math]x[/math] на [math]\alpha_n[/math]
  2. [math]S_n - [/math] элемент наилучшего приближения (в [math]\alpha_n[/math]) для [math]x[/math]
    [math]\| x - S_n \| = \inf_{y \in \alpha_n} {\|x - y} \|[/math]
  3. [math]\| S_n \| \leqslant \| x \|[/math]

Следствие:

[math]\displaystyle\sum |C_k(x)|^2 \times \| e_k \|^2 \leqslant \|x\|^2[/math] (Неравенство Бесселя)

Теорема Рисса — Фишера о сумме ряда Фурье. Равенство Парсеваля

Теорема:
[math]\{e_k\} - [/math] Ортогональная система в [math]\mathcal{H}, x \in \mathcal{H}[/math]
  1. Ряд Фурье [math]x[/math] сходится в [math]\mathcal{H}[/math]
  2. [math]x = \displaystyle\sum_{k=1}^{+\infty} C_k(x) \cdot e_k + z, [/math] тогда [math]\forall k \quad z \perp e_k[/math]
  3. [math]x = \displaystyle\sum_{k=1}^{+\infty} C_k(x) \times e_k \Leftrightarrow \displaystyle\sum_{k=1}^{+\infty} |C_k (x)|^2 \cdot \|e_k\|=\|x\|^2[/math] (Равенство Парсеваля)

Теорема о характеристике базиса

Теорема:
[math]\{e_k\}[/math] — ОС в [math]H[/math]. Тогда экв.:
  1. [math]\{e_k\}[/math] — базис
  2. Выполняется обобщённое уравнение замкнутости: [math]\langle x, y \rangle = \sum\limits_{k=1}^{+\infty} e_k(x) \overline{c_k(y))} \|e_k\|^2[/math]
  3. [math]\{e_k\}[/math] — замкнута
  4. [math]\{e_k\}[/math] — полная
  5. [math]Lin(e_1 e_2 \dots)[/math] — плотно в [math]H[/math]

Лемма о вычислении коэффициентов тригонометрического ряда

Теорема:
[math]T(x) - [/math] тригонометрический ряд, [math]\quad S_n(x) - [/math] частичные суммы

Пусть [math]f \in L^1[-\pi,\pi] \quad S_n \to f [/math] в пространстве [math]L^1[/math]

Тогда:

  1. [math]a_k = \dfrac{1}{\pi} \cdot \displaystyle\int\limits_{-\pi}^{\pi} {f(x) \cdot \cos {kx} \;dx}[/math]
  2. [math]b_k = \dfrac{1}{\pi} \cdot \displaystyle\int\limits_{-\pi}^{\pi} {f(x) \cdot \sin {kx} \;dx}[/math]
  3. [math]c_k = \dfrac{1}{2 \pi} \cdot \displaystyle\int\limits_{-\pi}^{\pi} {f(x) \cdot e^{-ikx} \;dx}[/math]

Теорема Римана — Лебега

Теорема:
[math]E \subset \mathbb{R}[/math] — измеримо, [math]f \in L^1(E)[/math]
Тогда [math]\displaystyle\int\limits_E {f(x) \cdot e^{ikx} \; dx} \xrightarrow[k \to \infty]{} 0[/math] (То же самое можно и с [math]\cos {x}[/math] и [math]\sin {x}[/math] вместо [math]e^{ikx}[/math])

Принцип локализации Римана

Теорема:
[math]f, g \in L^1[-\pi, \pi] \quad x_0 \in [-\pi, \pi] \quad \exists \delta \gt 0[/math]

[math]f(x) = g(x) [/math] при [math] x \in (x_0 - \delta, x_0 + \delta)[/math]

Тогда [math]S_n(f, x_0) - S_n(g, x_0) \xrightarrow[n \to +\infty]{} 0[/math]

Признак Дини. Следствия

Теорема:
[math]f \in L^1[-\pi, \pi] \quad x_0 \in [-\pi, \pi] \quad S \in \mathbb{R}[/math]

Пусть [math]\displaystyle\int_0^\pi \dfrac{|f(x_0+t)+f(x_0-t)-2S|}{t} \; dt \lt +\infty [/math]

Тогда [math]S_n(f, x_0) \xrightarrow[n \to +\infty]{} S[/math]

Корректность определения свертки

Теорема:

Свойства свертки функции из L^p с функцией из L^q

Теорема:
[math]f \in L^p \quad k \in L^q[-\pi, \pi] \quad \left(\dfrac{1}{p} + \dfrac{1}{q} = 1 \right) \quad 1 \leqslant p \lt +\infty[/math]

Тогда [math]f * k[/math] - непрерывна на [math][-\pi, \pi][/math]

[math]\|f * k \|_1 \leqslant \|f\|_p * \|k\|_q[/math]

Теорема о свойствах аппроксимативной единицы

Теорема:
[math]K_n[/math] — аппроксимативная единица.

Тогда [math](h \to h_0)[/math]:

  1. [math]f \in \widetilde{C}[-\pi, \pi] \quad f * K_n \operatorname*{\rightrightarrows}\limits_{h \to h_0} f[/math]
  2. [math]f \in L^p[-\pi, \pi] \quad \|f * K_n - f \|_p \to 0, h \to 0[/math]
  3. [math]f \in L^1, f[/math] — непр. [math]x_0 \quad K_n - [/math] ??? а.е.

[math]f * K_n[/math] — непрерывна в окрестности [math]x_0[/math]

[math](f * K_n)(x_0) \xrightarrow[h \to h_0]{} f(x_0)[/math]

Теорема Коши о перманентности метода средних арифметических

Теорема:
[math]\sum a_n = S \Rightarrow \sum a_n [/math] (по методу средних арифметических) [math] = S[/math]
Доказательство:
[math]\triangleright[/math]

[math]\sum a_n [/math] (по методу средних арифметических) [math] = \lim\limits_{n \rightarrow \infty} \frac 1{n + 1} \sum\limits_{k = 0}^n S_k[/math]

[math] \left|\dfrac{\sum_{k=0}^n S_k}{n+1} - S\right| = \left|\sum\limits_{k=0}^n \dfrac{S_k-S}{n+1}\right| \leqslant \sum\limits_{k=0}^{n} \dfrac{|S_k-S|}{n+1}\\ \forall \varepsilon \gt 0 \quad \exists N_1 \quad \forall n \gt N_1 \quad |S_n - S| \lt \dfrac{\varepsilon}{2} \\ \sum\limits_{k=0}^{N_1} \dfrac{|S_k-S|}{n+1} + \sum\limits_{k=N_1 + 1}^{n} \dfrac{|S_k-S|}{n+1} \lt \varepsilon[/math]
[math]\triangleleft[/math]

Теорема Фейера

Теорема:
3 пункта:
  1. [math] f \in \tilde{C}[-\pi, \pi] \Rightarrow \sigma_n(f, x) \operatorname*{\rightrightarrows}\limits_{n \to \infty} f(x)[/math]
  2. [math] f \in L^p[-\pi, \pi] \Rightarrow \|\sigma_n(f, x) - f \|_p \xrightarrow[n \to \infty]{} 0[/math]
  3. [math] f \in L^1, f - [/math] непр. [math] x \Rightarrow \sigma_n(f, x) \xrightarrow[n \to \infty]{} f(x)[/math]

Полнота тригонометрической системы

Теорема:
Тригонометрическая система полна в [math]L^2[/math] (Следствие теоремы Фейера)

Формула Грина

Теорема:
[math]\mathbb R^2[/math] — ориент. с помощью нумерации координат.

[math]D \subset \mathbb R^2[/math] — компактное, связное, односвязное, с [math]C^2[/math]-гладкой границей.
[math](P, Q)[/math] — гладкое векторное поле.
Пусть граница [math]D (\partial D)[/math] ориентирована согласованно с ориентацией плоскости.

Тогда [math]\displaystyle\int_{\partial D} P \,dx + Q \,dy = \displaystyle\iint_D \left(\dfrac{\partial Q}{\partial x} - \dfrac{\partial P}{\partial y}\right) dx\, dy[/math]

Формула Стокса

Теорема:
[math]D \subset \mathbb R^3[/math] — простая гладкая поверхность в [math]\mathbb R^3[/math],

[math]\partial D[/math][math]C^2[/math]-гладкая кривая,
[math]n_0[/math] — сторона поверхности; ориентированы согласованно с [math]\partial D[/math]
[math](P,Q,R)[/math] — гладкое векторное поле на [math]D[/math]. Тогда:

[math]\displaystyle\int_{\partial D} P dx + Q dy + R dz = \displaystyle\iint_D (R'_y - Q'_z) \;dy dz + (P'_z - R'_x) \;dz dx + (Q'_x - P'_y) \;dx dy[/math]

Формула Гаусса — Остроградского

Теорема:
[math]D \subset \mathbb R^3 \quad \partial D[/math] — ориент. полем внешних нормалей,

[math](P, Q, R)[/math] — гл. век. поле в [math]D[/math]. Тогда

[math]\displaystyle\iint\limits_{\partial D} P \,dy\,dz + Q \,dz\,dx + R \,dx\,dy = \iiint\limits_D (P'_x + Q'_y + R'_z)\,dx\,dy\,dz[/math]

Бескоординатное определение ротора

Теорема:

Бескоординатное определение дивергенции

Теорема:

Описание соленоидальных полей в терминах дивергенции

Теорема: